SM: Practice Board Exam #5

Lakukan tugas rumah & ujian kamu dengan baik sekarang menggunakan Quizwiz!

A 29 year old female patient presents with continued issues with both severe photosensitivity and a malar rash present across the face that does not encroach on the nasolabial folds. You suspect a possible diagnosis of systemic lupus erythematosus. Which of the following diagnostic tests is most specific to this diagnosis?

Correct Answer: Antinuclear antibody test (ANA) Rationale: While there is not one specific test to confirm a diagnosis of systemic lupus erythematosus, the antinuclear antibody test can be used in conjunction with current symptoms to establish this diagnosis. The ANA test is highly sensitive for this diagnosis.

A 41 year old woman is asking why her pain medication is not working as well as it should. The provider understands that drugs metabolized by the CYP2C19 enzyme...

Correct Answer: Are less effective in Asians Rationale: Asians can be poor metabolizers of the CYP2C19 enzyme. Therefore, drugs like pain medication and even omeprazole that are metabolized by this enzyme, can be less effective in this patient population. Patients can undergo genotyping to assess for this.

After obtaining a thorough history from a patient, the nurse practitioner is suspicious of possible papilledema. Which statement made by the patient may have led the nurse practitioner to this diagnosis?

Correct Answer: "While getting ready for work this morning, I began to experience flickering vision that lasted only a few seconds." Rationale: Papilledema is described as a sudden onset of vision changes including blurred vision or double vision, flickering, or loss of vision lasting seconds at a time. Accompanying symptoms caused by intracranial pressure may include a severe headache, nausea, and vomiting. This is an emergency as it can indicate increased intracranial pressure or a dangerously high blood pressure.

A patient calls the clinic with concerns over his upcoming inguinal hernia surgery. He has been taking Aspirin (Bayer) for the last several months, and cannot recall the surgeon's instructions on if he should discontinue this medication before or after surgery. Which of the following is the most appropriate response to this patient?

Correct Answer: "You will need to discontinue your daily Aspirin 7-10 days prior to the procedure." Rationale: Acetylsalicylic acid (Aspirin) has an irreversible effect on platelet aggregation that lasts the duration of the lifetime of the platelet (around 10 days). So, prior to surgery, patients should be advised to discontinue their Aspirin regimen for 7-10 days to help reduce the risk of bleeding during surgery

A 12 year old female patient presents with her mother to the clinic as her mother is concerned about the C-shaped appearance of her daughter's spine. The degree of curvature in the patient's spine is 35 degrees. Which of the following is appropriate to educate the mother about?

Correct Answer: "Your daughter's spine has moderate curvature, so we will need to place her in a Boston brace as treatment." Rationale: A curvature of 25-40 degrees is considered moderate scoliosis and in most cases requires bracing for correction. A curvature of 20 degrees or less indicates that the practitioner should wait and see if further intervention is needed at a later date. A curvature of greater than 40 degrees requires bracing and potentially surgery depending on the patient's risk.

Which of the following blood pressures would be considered stage 2 hypertension according to the AHA/ACC guidelines?

Correct Answer: 138/91 mmHg Rationale: According to the AHA/ACC (American Heart Association and American College of Cardiology) guidelines,.a blood pressure of 120-129/>80 mmHg would be considered an elevated blood pressure. Stage 1 hypertension is a blood pressure of 130-139/80-89 mmHg and we would initiate antihypertensives if the patient's ASCVD risk score was 10% or greater. Stage 2 hypertension is a systolic pressure ≥140 or a diastolic pressure >90 mmHg. The goal blood pressure is >130/80 mmHg with the AHA/ACC guidelines.

The nurse practitioner sees a patient today who complains of painless, localized eyelid swelling for the last 48 hours. Upon examination, a small rubbery nodule is noted. What is the most appropriate diagnosis and plan for this patient?

Correct Answer: Chalazion. Treat with warm compresses. Rationale: A chalazion presents as painless, localized eye swelling. A distinguishing factor between a chalazion and hordeolum, aside from these being painless, is the lack of erythema with chalazion. Treatment for chalazion includes applying moist, warm compresses to the area several times a day.

Which of the following is considered an emancipated minor? Select all that apply by choosing two of the following answer choices.

Correct Answer: 15 year old that is legally married / 17 year old who just enlisted Rationale: Emancipated minors are minors (<18 years old) that are legally married, active duty military, or have been emancipated by the courts. Although patients who are pregnant are not considered emancipated, they do not need parental consent for care related to pregnancy. Additionally, minors do not need parental consent for care regarding sexually transmitted infections or contraception.

A 66-year-old male patient comes to the clinic for a routine follow up appointment for his hypertension and diabetes. He describes feeling sad and worthless ever since losing his job 3 months ago. What is the next question the nurse practitioner should ask him?

Correct Answer: "Do you feel like you may harm yourself or others?" Rationale: The next question the nurse practitioner should ask the patient is about suicidal ideation. Asking about the risk of harm to themselves or others does not increase the chances of the patient taking action, but rather guides the provider on what to do next for the patient. Asking about starting an antidepressant or if the patient can afford therapy is inappropriate at this time without further evaluation. Assessing family and friend support is a good idea, but should not be asked next in this scenario.

An elderly male patient presents for a follow-up regarding a hospital admission for a transient ischemic attack two days prior. He states that he is confused by his diagnosis and would like further information. Which of the following statements would be inappropriate to teach this patient about in regards to this condition?

Correct Answer: "It is common with transient ischemic attacks to have some residual side effects. We will monitor you closely over the next few weeks." Rationale: While technically possible, it is extremely uncommon to have residual side effects after a transient ischemic attack (TIA). Traditionally, a true stroke has a much higher likelihood of leading to residual deficits. It is imperative that the patient is aware that this attack increases his chance of stroke in the future, especially in the first 90 days after the TIA.

The nurse practitioner has just diagnosed a patient with mononucleosis and understands that this diagnosis may put the adolescent at risk for the development of what cancer later in life?

Correct Answer: Hodgkin's lymphoma Rationale: Mononucleosis has been linked to the development of Hodgkin's lymphoma as well as Burkitt's lymphoma. While uncommon, this is due to the exposure of the epstein-barr virus.

A two year old presents to the clinic today with a nursemaid's elbow after swinging from his parent's arms earlier. The nurse practitioner performs a successful closed reduction utilizing the supination/flexion technique. Which of the following would be the most appropriate to tell the parents after the reduction?

Correct Answer: "After a successful closed reduction, there is no need to restrict his activity." Rationale: After having a successful closed reduction, the patient may resume their normal activities without need for any restrictions. This is the same for both reduction techniques - supination/flexion as well as hyperpronation/extension.Typically, children will not use the affected arm immediately after the reduction in anticipation of pain. Therefore, they are typically given 15-20 minutes to see if they can use the arm without pain and it may not happen instantaneously.

A 41 year old female patient was recently diagnosed with diabetes and has been taking metformin (Glucophage) for the last month. At today's visit, the patient states her friend with diabetes has started taking a daily Aspirin (Zorprin) and wonders if she should also be taking one. Which of the following statements is incorrect in regards to this patient's question?

Correct Answer: "Aspirin is routinely prescribed to all diabetic patients once they are 60 years old as the cardiovascular risk increases two fold at that time." Rationale: While Aspirin was once routinely prescribed to those with diabetes, this guidance changed by the American Diabetes Association in 2019. The current recommendation is that diabetic patients can receive Aspirin if they are at an increased cardiovascular risk, typically after the age of 50, as long as they receive education on the risks versus the benefits.

A 32 year old female patient was recently diagnosed with rheumatoid arthritis (RA) and started on methotrexate (Trexall). She researched the prescribed medication and would like to know why her rheumatologist gave her this drug at the onset of her disease. Which of the following statements is an appropriate response to the patient's question?

Correct Answer: "Disease-modifying antirheumatic drugs (DMARDs) are used first line for rheumatoid arthritis and started at the time of diagnosis to prevent rapid progression of the disease." Rationale: DMARDs are the first line treatment for rheumatology because they are immunosuppressive and immunomodulatory agents that work by broadly restricting the body's immune system. This mechanism of action slows disease progression by minimizing joint damage and provides symptom relief by reducing inflammation. Methotrexate (Trexall) is a frequently used DMARD, but contraindicated in pregnancy as it is Category X. Women of child-bearing age must be prescribed contraception before being prescribed this medication. Anti-TNF biologics are prescribed as a second line medication, or in combination therapy with DMARDs.

A 65 year old male presents for a medication reconciliation today. He reports taking Allopurinol (Aloprim), lisinopril (Prinivil), Atorvastatin (Lipitor) and acetaminophen (Tylenol) as needed. He has been doing well on these medications for the last three years. What medication education should the nurse practitioner provide at today's visit?

Correct Answer: "Please monitor for any onset of asymmetric facial swelling." Rationale: This patient needs to be thoroughly educated about the risk of angioedema while taking angiotensin-converting enzyme inhibitors (ACE) inhibitors. Even though this patient has been taking this medication without concern for three years, angioedema can present at any time. Signs and symptoms of angioedema include sudden or gradual onset of asymmetric facial swelling. If this occurs, the medication will be discontinued immediately. The patient will then be switched to another blood pressure medication such as an (angiotensin II receptor blocker) ARB. Although liver function, calcium, and blood glucose levels can be affected by the other medications this patient is taking, it would be more appropriate to obtain yearly relevant lab draws unless he is otherwise symptomatic.

A son and father come into your office today for the father's annual check up. The father is 94-years-old and has been recently diagnosed with prostate cancer. Other than the prostate cancer and hypertension, the father is healthy with no past medical history. The son with him today is his health care proxy and he is very upset that the oncologist did not want to initiate treatment. Which of the following is the best response by the nurse practitioner?

Correct Answer: "Prostate cancer is slow growing, so it is unlikely to ever affect your father's quality of life and treatment may make him feel sicker." Rationale: Prostate cancer is amongst the slowest growing types of cancer. Oftentimes, like in this scenario, when a patient is not expected to live longer than 10 years, providers opt out of treatment as it is not likely to add value or quality to the patient's lives. This is a delicate conversation, but explaining the reasoning behind certain practices, in a gentle manner, is extremely beneficial to provider-patient relationships.

A 10 year old female patient was recently diagnosed at an outside facility with pharyngitis and subsequently was prescribed Amoxicillin. A few days after starting the antibiotics, the patient develops an erythematous maculopapular rash with healthy skin intervening throughout. What should you educate the mother about?

Correct Answer: "Sometimes this rash occurs when patients are given antibiotics for viral etiologies of sore throat. It will resolve on its own." Rationale: The rash that this question is describing is a morbilliform rash. A morbilliform rash occurs when strep throat that is not bacterial in nature is treated with antibiotics. This can frequently occur when a patient with mononucleosis is treated with antibiotics such as Amoxicillin.

A three week old of Asian descent was brought into the clinic today by his parents due to concerns about the "coloring" of his back. The mother states that this new color variation started popping up in the last week or so. You note that during your physical exam there are several bluish-green spots over the lumbosacral area specifically. Which of the following is the most appropriate education to provide to the parents?

Correct Answer: "These are slate gray nevi, and will resolve on its own typically by 3-5 years of age." Rationale: "Slate gray nevus" is the medical term for a mongolian spot, which is the skin condition described in the question. These do typically resolve on their own and do not require referrals or further treatment.

A mother is concerned about her daughter's recent diagnosis of coarctation of the aorta and is seeking more information about the topic. Which of the following statements is incorrect in regards to this condition?

Correct Answer: "This condition will likely resolve spontaneously if the aortic narrowing is not severe." Rationale: This condition is due to a congenital defect. Coarctation of the aorta is typically found at birth if it is severe, and less frequently it is found later in life in those who have milder cases. Even in milder cases of coarctation of the aorta, it will not resolve spontaneously or on its own. It is common for these patients to receive surgical intervention including balloon angioplasty.

A mother presents with her 2-year-old son for a well-child check. As the nurse practitioner is completing the physical assessment, the mom voices concern over her son's knees pointing outwards when walking. Which of the following is the most appropriate response?

Correct Answer: "This is a common finding and will typically resolve on its own by the time he is 3-4 years old." Rationale: Genu Varum or "bow legs" is a common and normal finding in children. In this condition, the knees stay apart despite the child standing with their feet together. Typically, this condition resolves spontaneously by 3-4 years old and no intervention is necessary.

An 81-year-old patient presents to the office for his annual physical. At his last visit approximately six months ago, his lab work and physical exam were all within normal limits. During this visit, the nurse practitioner notes several yellow-white "lumps" around the inner part of the patient's eyelids bilaterally. The remainder of his physical exam is unremarkable. What should the nurse practitioner educate this patient about in regards to this finding?

Correct Answer: "This is a fairly common finding in women and men your age. It is benign in nature." Rationale: The patient is presenting with xanthelasma, which are simply cholesterol deposits around the patient's eye. When this condition presents in those who are young, it is oftentimes indicative of hyperlipidemia. When this condition presents in the elderly, it is typically a benign and chronic skin condition. Given his recent normal lab work and physical exam, it is appropriate to educate the patient about how this is benign in nature.

A 38 year old female patient was recently diagnosed with rheumatoid arthritis and started on Methotrexate (Trexall) by her rheumatologist. When providing patient education regarding the use of this drug, all of the following statements are to be included except:

Correct Answer: "Vitamin B can be purchased over-the-counter to take in conjunction with folic acid to enhance its effects." Rationale: Use of methotrexate (Trexall) causes the body to discrete folic acid in large amounts. Supplementation is necessary to prevent megaloblastic anemia, low platelets, and white blood cells because red blood cells cannot be produced with an insufficient amount of folic acid. More common symptoms include fatigue, paresthesias, headache, and muscle weakness. Vitamin B should not be used in combination with folic acid because it is a man made derivative of folate (vitamin B-9) and extreme levels can cause folic acid toxicity.

Which of the following patients would be the most appropriate to start on a SNRI (serotonin and norepinephrine reuptake inhibitor) for a new diagnosis of depression?

Correct Answer: A 33 year old female patient who is being treated for fibromyalgia with Gabapentin (Neurontin) Rationale: SNRIs are known to potentially increase blood pressure and therefore should be avoided in those with uncontrolled hypertension already. Concurrent use of the triptan drug class and SNRIs may lead to the development of serotonin syndrome. SNRIs are also processed in the liver, and if a patient has a liver condition, it may cause the drug to clear slowly. There is no contraindication for using Gabapentin (Neurontin) with a SNRI. SNRIs can be used to also help treat the neuropathic pain associated with fibromyalgia.

The nurse practitioner knows that all of the following patients would be appropriate to order a stool study for except:

Correct Answer: A 35 year old female patient with watery diarrhea after attending a cookout two days ago Rationale: The majority of gastroenteritis cases are viral in nature and therefore do not require a stool study to be performed. Stool studies are traditionally performed in those who are immunocompromised, have blood in the stool, present with a fever, or have had diarrhea for 10-14 days. The patient who attended a cookout likely has viral gastroenteritis that will resolve on its own.

A child presents to the office with complaints of hip pain. Which of the following assessment findings would support the diagnosis of Legg-Calve-Perthes disease over Slipped Capital Femoral Epiphysis or other hip pathology?

Correct Answer: A 5 year old who has had hip and knee pain for several weeks and the x-ray reveals necrosis of the femoral head Rationale: Legg-Calve-Perthes disease is characterized by osteonecrosis of the capital femoral epiphysis and occurs in prepubescent children. It may be accompanied by hip pain and a limp. Radiographs will reveal osteonecrosis of the femoral head in Legg-Calve-Perthes. The Trendeleburg test is performed by having a patient stand on one foot and assess if the pelvis stays horizontal or if it tilts. A positive Trendeleburg test (if a tilt is seen), is associated with both Legg-Calve-Perthes and slipped capital femoral epiphysis. Slipped capital femoral epiphysis is when the bone slips through the growth plate and occurs more often in growing adolescents. A widened physis will be seen on x-ray. Septic arthritis of the hip joint would cause hip pain and fever along with evidence of infection in the joint fluid.

Four patients call the after-hours clinic complaining of difficulties with their vision. Which of the four patients should the nurse practitioner instruct to go to the emergency room as soon as possible?

Correct Answer: A 65 year old female with a quick onset of eye pain, seeing halos and visual impairment Rationale: Sudden onset of severe eye pain, visual impairment and seeing halos is suspicious of acute angle closure glaucoma, which is a medical emergency to prevent permanent vision loss. A throbbing headache with sensitivity to light and preceded by floaters is likely a migraine with aura and not a medical emergency. Patients with Bell's palsy are at an increased risk for corneal abrasion and should always be prescribed eye lubrication for prophylaxis. Straight lines appearing wavy and rapid loss of central vision are key findings in wet age-related macular degeneration. While we do want to refer these patients to opthamology quickly, it does not need to be an immediate referral to the ER.

Which of the following is not considered to be a contraindication to taking an ACE-Inhibitor such as lisinopril (Prinvil)?

Correct Answer: A diabetic patient who has been unable to achieve adequate glycemic control despite being on two oral medications Rationale: Pregnancy, angioedema, and hyperkalemia beyond 5.5 mmol/L are all listed as true contraindications to take ACE-inhibitors. ACE-Inhibitors are encouraged in diabetic patients as long as their renal function remains mild to moderate. In patients with declining renal function, labs should frequently be monitored in case the medication needs to be stopped.

Which of the following patients would be considered at the lowest risk for the development of osteoporosis?

Correct Answer: A patient with a history of bulimia who went through menopause at age 49 Rationale: Each patient presented above has at least one risk factor for osteoporosis except for a woman with a history of bulimia who went through menopause at age 49. A history of anorexia puts our patient at risk for the development of osteoporosis. Other risk factors include older age, prolonged use of depo-provera (>2-5 years), early menopause (especially >45 years old), being a caucasian or Asian woman, history of smoking or alcoholism, long term use of Synthroid and PPIs, sedentary lifestyle, as well as low calcium and vitamin D intake.

The nurse practitioner knows that which of the following patients would likely be the worst candidate for the use of a thiazolidinedione (TZD) for treatment of their diabetes?

Correct Answer: A patient with a recent ejection fraction of 31% Rationale: The thiazolidinedione drug class has a black box warning for those with various types of heart failure. This black box warning is due to increased sodium and fluid retention, which can lead to serious exacerbations in these patients.

Which of the following patients would pioglitazone (Actos) not be the best medication choice to control their diabetes?

Correct Answer: A patient with active liver disease Rationale: Patients with active liver disease should avoid taking the thiazolidinediones (TZD) drug class for diabetes, like pioglitazone. Symptomatic heart failure, and NYHA class III and IV heart failure patients should also avoid the TZDs.TZDs may be used in conjunction with sulfonylureas, GLP-1s (glucagon like peptide I), SGLT2s (sodium glucose co-transporter 2), DPP-4s (Dipeptidyl-peptidase 4), and insulins.

A 64 year old diabetic patient has a new diagnosis of hyperlipidemia. The nurse practitioner anticipates starting a HMG-CoA reductase inhibitor such as rosuvastatin (Crestor) for this patient. What lab level will be essential to order prior to initiating this medication?

Correct Answer: AST/ALT Rationale: Ordering a liver function panel is necessary prior to starting a patient on a HMG-CoA reductase inhibitor. If the liver enzymes are elevated, the patient could be at risk for developing hepatotoxicity while taking this drug class.

A truck driver comes into the clinic for a diabetes check up. During the physical exam, the nurse practitioner notices a small, yellow, scaly lesion on his left arm. He states that it sometimes itches, but he has had it for so long that he mostly forgot about it. What is the nurse practitioner concerned about?

Correct Answer: Actinic keratosis Rationale: Actinic keratosis, a precursor to squamous cell carcinoma, presents usually as a yellow or tan (sometimes pink), scaly and raised lesion in a sun exposed area. These lesions may itch and burn and should be removed. They can be referred to skin or removed by fluorouracil (5FU) or cryotherapy.

An 8 year old presents to the clinic after three days of ear pain. Her mother states the pain was increasing in intensity each day until this morning when the child's ear began leaking serosanguinous and purulent drainage. The child reports that she is no longer in pain. The nurse practitioner suspects that which of the following is the most likely diagnosis in this patient?

Correct Answer: Acute suppurative otitis media Rationale: Given the patient's presenting history, it appears that the patient's tympanic membrane has ruptured. This can be referred to as suppurative otitis media when there is purulent drainage from the perforation itself. It is common for the tympanic membrane to recover within several weeks without additional intervention.

A 42 year old male with a past history of hypertension and diabetes is at the clinic today. He is currently managed with 2550mg metformin (Glucophage) per day and insulin glargine (Lantus) before bed. When reviewing his home blood sugar log, the nurse practitioner notices consistently high glucose levels every morning. When asked to check his blood sugar at 3 in the morning, his levels are consistently low. Which of the following treatment plans would be inappropriate?

Correct Answer: Add on a rapid acting insulin to take every morning Rationale: The Somogyi effect is a common cause of hyperglycemia in diabetic patients. The Somogyi effect is characterized by a drop in blood sugar around 2 or 3 in the morning which is then followed by rebound hyperglycemia upon awakening. When patients are experiencing the Somogyi effect, the goal of treatment is to prevent the dip, either by decreasing his insulin dose, adding a nighttime snack, or assessing if they have started any changes like exercising before bed that might be causing the 3 am hypoglycemia episode.

A 40 year old female of Asian descent presents with complaints of lethargy, nausea, diarrhea, and abdominal pain. She denies any recent excessive sun exposure but upon physical assessment you note hyperpigmentation of the lips and nipple area. You obtain labs and the results are as follows: Glucose: 115 mg/dL, Calcium: 9.2 mg/dL, Sodium: 128 mEq/dL, Albumin: 4.1g/dL, Potassium: 5.1 mmol/L, Creatinine: 1.1 mg/dL, BUN (blood urea nitrogen): 14 mg/dL. Based on this patient's symptoms and lab results, what diagnosis do you suspect?

Correct Answer: Addison's disease Rationale: Adrenal insufficiency, also known as Addison's disease, causes symptoms including fatigue, weight loss, nausea, vomiting, abdominal pain, myalgia, hypotension, and hyperpigmentation. The adrenal cortex, which is affected in a patient with Addison's disease, is in charge of producing cortisol and aldosterone. Aldosterone then regulates sodium retention and potassium excretion in the body. When this system is impaired in Addison's disease, it results in electrolyte abnormalities including hyperkalemia and hyponatremia, both of which were revealed in this patient's lab results.

In order to help confirm a suspicion of Cushing Syndrome, several diagnostic tests may be ordered. Which of the following diagnostic tests would be the most inappropriate to order for further investigation of this condition?

Correct Answer: Adrenocorticotropic hormone (ACTH) stimulation test Rationale: An ACTH test is typically used to help diagnose adrenal insufficiency which presents in cases of Addison's disease, not Cushing's disease or Cushing Syndrome. In Cushing's, there is too much cortisol present. Cortisol typically decreases at midnight, but will remain increased in a patient with this condition. Steroids such as Dexamethasone can be used to try to suppress cortisol secretion, but cortisol levels remain unchanged in Cushing's patients.

A 32 year old factory worker has been having progressive issues with dyspnea. The patient appears short of breath in the office and you note mild wheezing upon auscultation. There is no change in spirometry values pre and post administration of a bronchodilator. You suspect a possible airway condition such as COPD. What test would the nurse practitioner be most likely to order?

Correct Answer: Alpha-1 antitrypsin level Rationale: Abnormal values on spirometry often indicate problems with the airway such as asthma or chronic obstructive pulmonary disease. In asthma, the restriction is often reversible after administration of a bronchodilator. With COPD, the damage is already done and therefore the obstruction is unchanged. With COPD, our FEV1/FVC ratio will be decreased and any value >70% (or 0.7) is diagnostic. Whenever we have a younger patient diagnosed with COPD, we are suspicious of an alpha-1 antitrypsin deficiency. Ordering serum IgE allergen levels could be beneficial in a patient with severe allergies and asthma. A sweat chloride test is used to screen for cystic fibrosis - however, CF is most commonly diagnosed in the first few years of life.

A 63-year-old male comes into the clinic to discuss his medications and to review recent lab work. He tells the nurse practitioner that he is having pain that shoots down his left leg into his foot that sometimes makes his foot feel "numb and tingly." What medication can the nurse practitioner prescribe to help alleviate his symptoms?

Correct Answer: Amitriptyline (Elavil) Rationale: What the patient is describing is neuropathic pain - burning, shooting, tingling pain that can cause numbness and tenderness. Tricyclic antidepressants, such as amitriptyline (Elavil), can be beneficial in reducing these symptoms. Methotrexate (Trexall) is commonly used for conditions such as rheumatoid arthritis. Naproxen (Aleve), an NSAID, could help some with the pain, but amitriptyline would be a better medication to try. Phenelzine is a monoamine oxidase inhibitor (MAOI) antidepressant that does not help with pain.

A 28 year old female who is 22 weeks gestation reports concerns regarding antibiotic use during pregnancy. She was just diagnosed with strep throat at an urgent care and was prescribed amoxicillin (Amoxil), but is apprehensive to start the antibiotic. Which of the following statements is true regarding antibiotic use during pregnancy?

Correct Answer: Amoxicillin (Amoxil) is part of the penicillin drug class which is preferred during pregnancy due to its low side effect profile Rationale: Penicillin has multiple indications and is pregnancy Category B, meaning it is safe to take throughout pregnancy. In addition to penicillins, antibiotics from the cephalosporin and macrolide drug classes are also Pregnancy Category B at present time. If a penicillin allergy is present and there is no history of an IgE mediated response like anaphylaxis, it is appropriate to prescribe an antibiotic from the cephalosporin drug class. Sulfamethoxazole/trimethoprim (Bactrim) is pregnancy Category C and should generally be avoided in pregnancy.

A middle aged patient with heart failure was placed on Lasix (furosemide) daily after a recent exacerbation. Which of the following is not considered a potential side effect of this medication?

Correct Answer: Hypernatremia Rationale: Lasix (furosemide) is a loop diuretic that efficiently removes excess fluid from the body. In the removal of this fluid, it often inadvertently removes electrolytes as well. It is common to see issues with many electrolytes including but not limited to - hypokalemia, hypocalcemia, hypomagnesemia, and hyponatremia.

The nurse practitioner sees a patient in the clinic who is complaining of round lesions that tend to appear whenever she is stressed. The lesions are raised and have a silvery-white appearance. When assessing the area, a small piece falls off, and some bleeding is noted. What is this finding called?

Correct Answer: Auspitz Sign Rationale: Auspitz sign is a classic clinical finding in plaque psoriasis. It occurs when an area of plaque is removed and small, pinpoint bleeding occurs. Koebner phenomenon is also a sign of plaque psoriasis, but it is when the plaques form over an area of previous trauma.

The nurse practitioner is caring for a patient who has a terminal cancer diagnosis. The patient has expressed interest in stopping all treatment and moving to hospice care instead. The family pulls the provider aside and states that they would like his treatment to continue instead. The nurse practitioner understands that this would violate which ethical principle?

Correct Answer: Autonomy Rationale: Autonomy is the ability of the patient to make their own decisions. There is no indication that this patient is incompetent and therefore the patient has the right to decide what type of care he receives. Paternalism is what occurs when the family or a healthcare provider actually overrules a patient's wishes and violates their autonomy. Veracity is about truth telling. Nonmaleficence is about avoiding harm to patients.

A mother brings her 3 year old daughter to the office today due to concerns about her eyes. She notes that yesterday her daughter woke up with an extremely erythematous right eye and this morning both of her eyes were erythematous and painful. She has been informed by daycare that other toddlers in her classroom are having similar symptoms. She notes there has also been some mild drainage. Which of the following differentials would be most likely given this history? Select all that apply by choosing two of the following answer choices.

Correct Answer: Bacterial conjunctivitis / Viral conjunctivitis Rationale: Both bacterial and and viral conjunctivitis should be considered for this patient. Bacterial and viral are highly contagious, making her history of exposure at daycare pertinent information. In addition, both bacterial and viral conjunctivitis typically begin unilaterally and progress to bilateral symptoms within 24-48 hours. Further assessment of the drainage should be done to determine if this is bacterial or viral in nature. If bacterial, the drainage will likely be purulent whereas viral will present with a more string-like discharge.

Which act required nurse practitioners to acquire an NPI number in order to bill Medicare?

Correct Answer: Balanced Budget Act Rationale: The Balanced Budget Act of 1997 required that all nurse practitioners apply for an NPI number in order to bill medicare and medicaid. It also determined that nurse practitioners could bill at 85% of what a physician charges. The Budget Reconciliation Act of 1989 was the first law that allowed NPs to get paid by medicare. Which of the following cranial nerves, when infected with the herpes zoster (shingles) virus, may cause blindness?

All of the following are risk factors for peripheral arterial disease except

Correct Answer: Body mass index of 28 Rationale: A BMI of greater than 30 specifically is considered a risk factor for peripheral arterial disease. Diabetes mellitus, atherosclerosis, and cerebrovascular disease are all considered risk factors for this condition.

All of the following are possible assessment findings in a patient with bacterial endocarditis except:

Correct Answer: Bony enlargement of the proximal interphalangeal joint Rationale: Osler's nodes, roth's spots, and splinter hemorrhages are all possible assessment findings associated with a diagnosis of bacterial endocarditis. Painful, red-purple lesions on the hands and feets are commonly referred to as osler's nodes. Retinal hemorrhages with a white center are descriptive of roth's spots. Red-brown lines underneath the fingernails is indicative of a splitter hemorrhage. Bony enlargement of the proximal interphalangeal joint is known as bouchard's nodes, which are seen mostly common with conditions such as osteoarthritis.

A 47 year old premenopausal female patient reports to the clinic today with complaints of new breast leakage. She states that she has not had breast leakage since she breastfed her last child almost fifteen years ago. You suspect a potential diagnosis of galactorrhea as her physical exam is benign. Which of the following tests would be inappropriate for further confirmation of this diagnosis?

Correct Answer: Breast ultrasound Rationale: Primary hypothyroidism can lead to both hyperprolactinemia and galactorrhea simultaneously. Prolactinomas are another potential cause of galactorrhea. A pregnancy test would be appropriate as she is premenopausal. A breast ultrasound or mammogram could be utilized if this was suspected to be pathologic in nature, but given the benign physical exam that is unlikely.

Which of the following cranial nerves, when infected with the herpes zoster (shingles) virus, may cause blindness?

Correct Answer: CN V Rationale: Cranial nerve 5, the trigeminal nerve, has an ophthalmic branch that when damaged by the herpes zoster (shingles) virus, can lead to permanent blindness. Because of this, if a patient with shingles presents with any lesions on their face, especially near their eye(s), they need to be referred to ophthalmology for management.

Treatment and management of osteoarthritis can include all of the following except?

Correct Answer: Calcium Supplements Rationale: Calcium supplements are not indicated in the treatment and management of Osteoarthritis. These supplements would be indicated in osteoporosis instead. Weight bearing exercise, smoking cessation, and glucosamine have all been shown to help improve pain and joint function associated with the disease.

At birth, infants are subjected to a variety of newborn screening tests. One such test is a screening for congenital hypothyroidism. Which of the following is not considered a possible long term effect of untreated congenital hypothyroidism?

Correct Answer: Caput succedaneum Rationale: Caput succedaneum is when the scalp swells in a newborn, typically due to pressure from a vaginal delivery. Untreated congenital hypothyroidism leads to a slowing of growth all throughout the body including mental, sexual, and physical development.

In the elderly, vascular dementia is oftentimes related to a previous history of which of the following?

Correct Answer: Cerebrovascular accident Rationale: Vascular dementia is the second most common form of dementia and typically results when there is a condition in which there is a lack of adequate blood flow to the patient's brain. The typical cause of vascular dementia is related to a stroke, also known as a cerebrovascular accident.

When creating content to be distributed to parents of kindergartners, which of the following would be important to include with regards to bowel movements and functional constipation?

Correct Answer: Children should be encouraged to sit on the toilet as soon as they feel as if they need to defecate Rationale: Functional constipation must meet two or more of the abnormal following criteria: history of withholding stool, history of painful or large bowel movements, history of large diameter stool, presence of a large fecal mass in the abdomen, two or fewer defecations in the toilet per week, and at least one episode of fecal incontinence per week. A KUB x-ray may be ordered if functional constipation is suspected, but is not always necessary. Children should be encouraged to regularly sit on the toilet to help prevent constipation.

Which of the following professions is not considered a physician under the state nursing practice act, and therefore cannot participate in a collaborative agreement?

Correct Answer: Chiropractor Rationale: Chiropractors are not eligible to participate in a collaborative agreement at this time. This does not mean that nurse practitioners and chiropractors cannot work together in other capacities.

A middle aged patient presents to the clinic for a follow up after requiring lithotripsy for several kidney stones. When reviewing the report from the urologist, it is noted that the patient had stones composed of calcium oxalate specifically. Which of the following foods should the patient be educated to avoid to ideally prevent these stones from occuring again? Select all that apply by choosing four of the following answer choices.

Correct Answer: Chocolate / Spinach / Rhubarb / Tangerines Rationale: Calcium oxalate stones are the most common type of kidney stone. When a patient consumes too many foods that are high in oxalate, it is difficult for the kidneys to filter it out efficiently which leads to the oxalate binding with calcium. This binding then leads to the formation of kidney stones. Foods high in oxalate include items such as chocolate, spinach, rhubarb, beans, tangerines, coffee, cranberries, etc.

Which of the following antibiotics puts patients at the greatest risk for developing Clostridium difficile?

Correct Answer: Clindamycin (Cleocin) Rationale: Exposure to antibiotics is the biggest risk factor for developing Clostridium difficile (C. diff) infections. The most notorious antibiotics for causing C. diff infections include clindamycin (Cleocin), cephalosporins, and fluoroquinolones. C. diff infections are typically treated with vancomycin (Vancocin).

An 81 year old diabetic patient presents for follow up today for a non-healing ulcer on his right foot. Upon evaluation of the wound, you note red streaking away from the area into the ankle and lower leg. There is a foul smelling purulent drainage coming from the ulcer as well and when questioned, the patient states "it has been getting worse over the last few days". What is the most appropriate treatment for this patient?

Correct Answer: Clindamycin (Cleocin) Rationale: The most appropriate treatment for this patient is clindamycin (Cleocin) as cellulitis is the most likely diagnosis. This patient's wound and diabetes put him at risk for an infection. Cellulitis commonly presents as redness, warmth, and edema and frequently affects the lower extremities. Due to the presence of purulent drainage, this patient specifically has purulent cellulitis making cephalexin (Keflex) an inappropriate choice.

A patient comes in with atopic dermatitis on the underside of his chin. He states it is extremely itchy and the nurse practitioner notes some weeping erythematous skin on exam. Which of the following topical steroid options would be the least appropriate to prescribe to this patient?

Correct Answer: Clobetasol (Temovate) Rationale: Knowing the basics of topical steroid treatment is important for practice and the exam. The low potency steroids like low percentage hydrocortisone and alclometasone dipropionate are suitable options for the face as they are Class 7 and 6 steroids, respectively. High potency steroids, like clobetasol, should never be used on the face as they can cause skin sloughing.

A newborn's blood pressure is checked in the nursery at the hospital. Which of the following is a potential diagnosis that can come from this assessment?

Correct Answer: Coarctation of the aorta Rationale: Coarctation of the aorta can be suspected at birth by taking a blood pressure in the upper and lower extremities in a newborn. An oxygen reading above and below the heart can also provide clues that an infant might have coarctation of the aorta. This is a serious heart defect that needs a cardiology referral as soon as possible and is extremely important to screen for.

A 43-year-old female presents to the office with complaints of joint pain for the last several months. The pain is mainly in her wrists and knees. She also reports that sometimes she just feels "sore all over" and it is always worse in the morning but still persists throughout the day. What labs would be inappropriate for the nurse practitioner to order to help confirm the diagnosis?

Correct Answer: Comprehensive Metabolic Panel (CMP) Rationale: Sedimentation rate, rheumatoid factor, and complete blood count are all lab tests that would be ordered with suspected rheumatoid arthritis; sedimentation rate would be elevated due to inflammation. The CBC often shows micro or normocytic anemia; the RF factor is positive in 3 out of 4 patients.

The nurse practitioner is evaluating a 25 year old male and notes a single, skin-colored, cauliflower-like lesion on his penis. He denies associated pain. Upon further examination, the lesion is soft and 5 mm in diameter. Which of the following is the most likely diagnosis?

Correct Answer: Condylomata acuminata Rationale: Condylomata acuminata (anogenital warts) may present as single lesions or in groups. They range from skin-colored, to erythematous, to brown and may be flat, dome-shaped, cauliflower-like, or plaque-like. They are typically asymptomatic, consistent with this patient's experience. Molluscum contagiosum commonly present as umbilicated papules while syphilis commonly presents as a nontender, indurated lesion. Bowenoid papulosis, a premalignant condition, presents as several red papules in the gential area.

The nurse practitioner is attending a pediatric conference. The teaching over infant mortality has been effective when the nurse practitioner identifies what as the leading cause of death in infants?

Correct Answer: Congenital malformations Rationale: Congenital malformations such as congenital heart defects or chromosomal abnormalities are the most common cause of death in infants. The most common cause of death in children is accidents or unintentional injuries such as drowning or motor vehicle accidents.

A 68-year-old female patient comes into your office. She states her lower leg had been hurting her, so she used a heating pad the night before. This morning, she woke up with a red rash in the area the pad was. It does not extend past where the pad had been on her skin, and is clearly demarcated. Which of the following would be the most appropriate diagnosis?

Correct Answer: Contact dermatitis Rationale: This is a classic case of contact dermatitis. Because the patient's rash is well demarcated, in the area where a known contact (the heating pad) occurred, we can be confident that this was the cause. If the rash extended past where the heating pad was, we may consider shingles as a possible differential. The rash should resolve on its own as long as the heating pad is no longer used, but if not, a 1-2 week course of topical steroids can also be prescribed.

During a fundoscopic exam, the nurse practitioner notes white, fluffy appearing spots in the patient's eye. Which of the following is the correct term for these spots and which diagnosis are they indicative of in the patient?

Correct Answer: Cotton wool spots, diabetic retinopathy Rationale: Fluffy yellow-white spots seen on a fundoscopic exam are referred to as "cotton wool spots" and are a common finding with diabetic retinopathy. If these are seen on exam, it warrants further investigation into the patient's glucose status and control.

A 65 year old female presents to the office today with concerns about a new mark on her back. She states she has never had skin concerns until this time and only notices it when her shirt rubs the area, causing it to be mildly pruritic. Upon examination, you note a black well-demarcated, oval lesion with a stuck-on appearance. She is requesting that it be removed because it constantly rubs against her shirt and is distracting. Which of the following is the best plan for this patient?

Correct Answer: Cryotherapy Rationale: The lesion being described is consistent with seborrheic keratosis. These lesions are well-demarcated, can be round or oval in shape, and can vary in color from skin colored all the way to black. They have the classic stuck-on appearance and are usually asymptomatic, but can be painful, bleed, or cause itching with any friction or irritation. Since these are benign and slow growing, they do not generally require any treatment however if bothersome to the patient can be treated with cryotherapy, shave excision, or electrodesiccation.

You are seeing a 26 year old patient who is 18 weeks pregnant. She is asking for more information regarding her alpha fetoprotein (AFP) results. What education is true regarding AFP test results?

Correct Answer: Decreased alpha fetoprotein (AFP) levels may indicate that baby has Down Syndrome Rationale: Alpha fetoprotein (AFP) testing is completed between 16 and 20 weeks pregnancy. Results, if abnormal, can help detect neural tube defects as well as different types of chromosome abnormalities. Low AFP levels may indicate chromosome abnormalities such as Trisomy 21 or Trisomy 18.

An elderly patient presents to the clinic today with complaints of increased sneezing, coughing, nasal itching, and nasal drainage. He also reports that his eyes have been burning and tearing up more than normal. Upon further discussion and evaluation, you diagnose this patient with allergic rhinitis. Which medication commonly used to treat allergic rhinitis should be avoided in this patient?

Correct Answer: Diphenhydramine (Benadryl) Rationale: Antihistamines, especially older generation antihistamines such as diphenhydramine (Benadryl) are a common adjunct therapy for allergic rhinitis but should be avoided in the elderly due to the anticholinergic side effects.

Signs of Duchenne's muscular dystrophy commonly include all of the following except:

Correct Answer: Diplopia Rationale: Duchenne's muscular dystrophy is a condition in which the muscles progressively weaker over time. Beyond muscle weakness, Duchenne's can impact almost every area of life including their cognitive function. Large calf muscles are normal in these children due to scar tissue build up from muscle break down. Due to proximal weakness as well, they also progress to eventually having a waddling gait. Diplopia is unlikely to be seen with Duchenne's muscular dystrophy and instead would be more likely to be seen with oculopharyngeal muscular dystrophy.

In patients with celiac disease, it is common that the physical exam will be mostly benign and therefore it can be difficult to diagnose. Which of the following comorbidities might warrant celiac disease screening in an asymptomatic patient? Select all that apply by choosing three of the following answer choices.

Correct Answer: Down syndrome / Selective IgA deficiency /Turner's syndrome Rationale: According to the North American Society for Gastroenterology, Hepatology and Nutrition guidelines, it is recommended to screen asymptomatic patients for celiac disease if the individual has a first degree relative with the disease, type 1 diabetes, down syndrome, thyroiditis, Turner's syndrome, and selective IgA deficiency.

All of the following are possible outpatient treatment options for community acquired pneumonia for patients with significant comorbidities except:

Correct Answer: Doxycycline (Vibramycin) Rationale: According to the 2019 guidelines published by the American Thoracic Society and the Infectious Disease Society of America, there are two treatment options for outpatient treatment of community acquired pneumonia in adults with comorbidities. Combination therapy (Augmentin in addition to a macrolide such as Azithromycin) or monotherapy (a respiratory fluoroquinolone) may be utilized. Doxycycline can be used as treatment for community acquired pneumonia, but is not a preferred treatment for those with comorbidities such as renal disease, heart disease, diabetes, etc.

A 68 year old male is at the office following a recent visit to the emergency room. While walking into a store, he had tripped and outstretched both arms in an attempt to ease his fall. He is now experiencing pain with overhead movement that worsens at night. Which assessment test would be most appropriate to perform on this patient?

Correct Answer: Drop Arm Test Rationale: Signs and symptoms of a torn rotator cuff most often include weakness and pain with overhead movement. Pain is most prevalent at night and may cause sleep disturbances. Clicking, crepitus, and limited range of motion are common objective findings. The drop arm test is used to assess for rotator cuff injuries. The examiner grasps the patient's wrist and abducts the shoulder to 90 degrees. Once the patient's arm is released, they are instructed to slowly lower their arm. The test is positive if the patient has little control while lowering their arm.

You are seeing a 19 year old athlete in the office today for right shoulder pain and weakness. He notes that he plays football and believes he injured it at his game last week. Upon examination, his pain is over the lateral deltoid and he reports that overhead activity worsens his pain. What is the most appropriate test to perform in the office today for further assessment of this injury?

Correct Answer: Drop arm test Rationale: Of the tests listed, the drop arm test is the only one that assesses rotator cuff tears. It is performed by passively abducting the shoulder and asking the patient to slowly lower their arm back down. If a tear is present, the arm will drop to the side. Both Neer's and Hawkins test are assessing for impingement syndromes while O'Briens test is looking for a SLAP (superior labrum anterior posterior) tear.

An elderly Muslim female comes into the office for evaluation of potential skin cancer and an overall skin check. Upon exam, the nurse practitioner notices that her skin looks tented and she seems mildly dehydrated. The patient is offered water during the exam, but she refuses and states that it is Ramadan so she must wait until sundown to eat or drink. What is the most appropriate way to proceed with this visit?

Correct Answer: Educate her on the physical findings you are seeing that indicate possible dehydration and document this in your note Rationale: During Ramadan, people of Muslim faith do not eat or drink from sunrise to sunset. This is a 30 day period, where all oral consumption must be done during the sundown to sunrise time frame. Although this patient may be dehydrated, they still have autonomy and can make decisions for themselves. Findings should be documented. IV hydration is not indicated in cases of mild dehydration

A 25-year-old female comes into the office with concerns about a sore that appeared on the vermillion border of her lip four days ago. She reports that initially it was just an erythematous "bump" but it has progressively gotten worse. During the exam, a fluid-filled blister is noted that is starting to crust over. Which of the following would be the most appropriate treatment to prescribe to this patient?

Correct Answer: Educate the patient she is not a good candidate at this time for antiviral medications Rationale: The patient is experiencing HSV1. Treatment for HSV is typically antiviral medication within the first 48-72 hours. Because this patient is on day 4 (96 hours) of having the sore, it would be too late to initiate antivirals. It is important to educate this patient to use over-the-counter topical anesthetics or non-steroidal anti inflammatory drugs (NSAIDs) for pain relief.

A 30-year-old female patient comes to the clinic for her yearly examination. She states that she read online that "the pill" can help reduce the chances of certain cancers. The nurse practitioner is aware that there is a decreased risk of which of the following cancers as a result of taking combined oral contraceptives:

Correct Answer: Endometrial and ovarian cancer Rationale: Studies have found that women who have an increased risk of endometrial and ovarian cancer can have their chances reduced when taking a combined oral contraceptive. There is no supportive data at this time that combined oral contraceptives can reduce the risk of cervical or breast cancer.

A mother brings her 7 year old child into the clinic with the chief complaint of a fever for 4 days. She reports that the child's fever has been resolved since yesterday, but wanted to still bring her child in to get looked at. Upon exam, the nurse practitioner notes a lacy, maculopapular rash along the arms and trunk and very flushed cheeks. What condition does the nurse practitioner suspect?

Correct Answer: Erythema infectiosum Rationale: Based on this characteristic description of a fever that resolves followed by the lacy rash and "slapped-cheeks," this child has erythema infectiosum, also known as Fifth Disease This is caused by Parvovirus B19.

The nurse practitioner is teaching a young adult patient about how to prevent a urinary tract infection through lifestyle modifications. Lifestyle modifications are imperative in preventing future urinary tract infections. The nurse practitioner knows this is because which of the following bacteria is the leading cause?

Correct Answer: Escherichia coli Rationale: Escherichia coli (E. coli) causes almost ninety percent of all urinary tract infections. This is due to the fact that E. coli is commonly present in the feces. This leads to easy contamination of the urinary tract if the patient has poor hygiene practices.

A 12 year old patient is presenting with shortness of breath, cough, and wheezing. Which of the following assessment findings are most consistent with a diagnosis of moderate persistent asthma?

Correct Answer: FEV1 66% with daily symptoms Rationale: When a FEV1 is between 60-80%, the patient is considered to be in moderate persistent asthma. When the FEV1 is greater than 80%, the patient may have intermittent or mild asthma depending on their symptom severity.

You are initiating the Gardasil vaccine in an 11 year old male. Which of the following would not be common side effects of this vaccine to educate the patient and his parents about?

Correct Answer: Fatigue and mild rash Rationale: The Gardasil vaccine may cause mild fatigue, but should not typically cause any type of rash. Fever, sore arm, muscle and joint pain, and dizziness and syncope are all common side effects of this vaccine. We should instruct patients that they can take tylenol after as needed. Additionally, to lower the risk of syncope, we want to administer this while they are sitting or laying down and keep them sitting down in the office for 10-15 minutes after administration.

Which of the following clinical signs or symptoms could help a nurse practitioner distinguish between a diagnosis of osteoarthritis or rheumatoid arthritis?

Correct Answer: Fever Rationale: Joint stiffness and pain, Bouchard's nodes, and increased intensity of symptoms upon awakening can all be present in both rheumatoid arthritis and osteoarthritis. However, rheumatoid arthritis is the only one that presents with potential systemic symptoms such as fever.

A mother and her 10 year old son are at the clinic today for his annual checkup. The mother is asking the nurse practitioner when puberty is expected to start in her son. The nurse practitioner understands that puberty starts in which tanner stage?

Correct Answer: II Rationale: The start of puberty in both males and females starts with the beginning of tanner stage II. In tanner stage II, males get a few, fine pubic hairs and enlargement of the scrotum. Tanner stage III is the stage in which the penis elongates the most. In tanner stage IV, the penis grows in width and the scrotum/testes enlarge and darken.

A father brings his 3 year old daughter in today for a sick visit. He reports that for the last 3 days she has had a fever of 101.3 to 102.4, a runny nose, and red rash across her face. Today upon awakening, he noted a rash on her chest and the soles of her feet. He has been managing at home with children's acetaminophen (Tylenol) and ibuprofen (Motrin), but brought her into the clinic today as she does not seem to be improving. Which of the following is the most likely diagnosis for this patient?

Correct Answer: Fifths disease Rationale: Fifths disease (also known as erythema infectiosum), caused by parvovirus B19, is more common in children than adults. It typically begins with a fever and progresses into the development of a rash. The rash classically appears as a slapped cheek rash on the face and a lacy net like rash on the body. This dad should be informed that since the rash has appeared, she is no longer contagious and he can continue treating the fever with acetaminophen (Tylenol) and ibuprofen (Motrin) at home and ensure she is receiving adequate hydration and rest.

Which of the following supplement(s) should a newly pregnant woman be taking every day?

Correct Answer: Folic acid Rationale: Folic acid is an important supplement during pregnancy to ensure the developing fetus develops normally neurologically. A complete prenatal multivitamin is also important during pregnancy to make sure the mother is able to supply all necessary nutrients to the growing fetus. Ideally, folic acid supplements and prenatal vitamins would be started 2-3 months before conception.

The nurse practitioner is assessing a child whose legs resemble the picture above. What is the likely differential diagnosis?

Correct Answer: Genu Valgum Rationale: Source: https://commons.wikimedia.org/wiki/File:Genu_valgum.svg This is genu valgum, also known as knock knees. It is as if there is "gum" sticking the knees together. Genu varum is known as bow legged.

A 18-year-old female patient presents for follow up after an emergency room visit. She reports that she had diarrhea, fever, severe stomach cramping due to gas pain, and no appetite. She tells you that they took a sample of her stool, gave her intravenous fluids, and sent her home with a prescription for Flagyl (Metronidazole). What condition was this patient most likely treated for?

Correct Answer: Giardia Rationale: Treatment for Giardia is Flagyl. Giardia lamblia is a protozoan with flagella that can infect the small intestine and is commonly seen in those who have been to rural areas or around others (humans or pets) that have been affected. Symptoms can last 2-4 weeks and include nausea, diarrhea, gas, abdominal pain, and decrease in appetite. To diagnose this condition, a stool sample should be collected and examined in the lab for the presence of the parasites.

Which of the following labs is most specific to the kidneys?

Correct Answer: Glomerular filtration rate (GFR) is most specific to the kidneys because it measures the blood that is passed through the glomeruli every minute Rationale: GFR is the most specific test to the kidneys, and for this reason, it is used to stage chronic kidney disease (CKD). CKD is staged off of the worsening glomerular filtration rate. Remember, a normal GFR is >90 mL/min. Stage I has a GFR still >90 mL/min, but there is kidney damage, Stage II is 60-89mL/min, Stage III is 30-59 mL/min, Stage IV is 15-29 mL/min, and Stage V, or end stage, is a GFR < 15mL/min. To diagnose CKD, the GFR needs to be >60 mL/min for 3 months or longer.

In a young adult male patient with increasing skin hyperpigmentation and worsening fatigue, a diagnosis of Addison's disease is suspected. Which of the following labs would be unlikely to see in a patient with this condition?

Correct Answer: Glucose 134 mg/dL Rationale: In patients with Addison's disease, it is common to see issues with hyponatremia, hyperkalemia, and increases in blood urea nitrogen (BUN). It is more typical to see issues with hypoglycemia than hyperglycemia, and their glucose levels can decrease to below 50 mg/dL inadvertently.

A 37-year-old female patient comes into your office's free sexual health clinic. She states he has been having on and off fevers, night sweats, unintentional weight loss, and general malaise for a few weeks now. She has a sore throat today and painless lymphadenopathy. When you inquire about her sexual history she states she frequently has unprotected sex with both males and females, but states she was tested for "everything" a few years ago. What diagnostic tests would be most critical to run today?

Correct Answer: HIV-1/HIV-2 antibody antigen assay test Rationale: This patient is exhibiting signs of an acute HIV infection. The fever, malaise, and unintentional weight loss coupled with the varied sexual history are good indications this patient has HIV. It would be most critical to use a HIV-1/HIV-2 antibody antigen assay test, as well as a CD4 T-cell count to be able to quickly diagnose and stage this patient. Nucleic acid amplification tests can be used to detect syphilis and chlamydia, which would be important to assess in this patient as well. A pap-smear would be indicated if she was not up to date with them, and KOH prep tests are used to test for yeast infections, of which this patient does not have symptoms.

The nurse practitioner conducts a snellen chart test on an 18-year-old male patient. The results for his left eye is 20/40 and his right eye is 20/20. Which of the following is the correct interpretation of these results?

Correct Answer: He sees in his left eye at 20 feet what a normal person sees at 40 feet, and his right eye has perfect vision Rationale: The Snellen chart is used to test visual acuity. The top number, 20, is the distance the patient stands away from the chart on the wall, and never changes. The bottom number, in this case, 40 on the left, and 20 on the right, tells us what the patient can see in comparison to what a normal person (20/20) can see. Because this patient is 20/40 in his left eye, this means that he sees at 20 feet what a normal person sees at 40 feet. His right eye has perfect 20/20 vision.

A young female patient presents with severe abdominal pain and abnormal vaginal bleeding. Which of the following signs and symptoms would be suggestive of a possible ruptured ectopic pregnancy? Select all that apply by choosing three of the following answer choices.

Correct Answer: Heart rate 139 / Pain that occurs after palpation / Bluish discoloration Rationale: Both clonus (involuntary muscle contractions of the hands, ankle, jaw, etc) and proteinuria typically seen in patients suffering from severe pre-eclampsia. Tachycardia as well as hypotension are likely to be seen in a ruptured ectopic pregnancy due to fluid volume loss. In addition, A positive blumberg sign (rebound tenderness) is possible in a rupture due to the fact the rupture can lead to peritonitis. Cullen's sign may be present due to the intra-abdominal bleeding that would occur after rupture.

A three year old presents to the clinic today with her mother. She states that the child had this new rash "pop up" after she recovered from an upper respiratory infection approximately a week and a half ago. On the physical exam, you note petechiae on her lower legs as well as some mild bleeding in her gums. Otherwise, the exam is benign and you also note there is no splenomegaly present. What is the most likely cause of these symptoms?

Correct Answer: Immune Thrombocytopenia (ITP) Rationale: The key factor in this question is that the child is recovering from a recent respiratory infection. The average age of diagnosis for Immune Thrombocytopenia in children is two to six ears old. It is also noted specifically that there is no splenomegaly which is characteristic of ITP.

Approximately one week ago, you diagnosed a middle aged patient with COPD with an acute upper respiratory infection. He returns to the clinic today complaining of increasing symptoms, and you diagnose him with a COPD exacerbation. Which of the following criteria is not an indication to start this patient on antibiotics?

Correct Answer: Increased cough Rationale: An increased cough in a COPD patient is not an indication to start antibiotics, as they frequently have flare ups. Additionally, the majority of upper respiratory infections are self-limiting as they are viral. Increased dyspnea, sputum volume, and/or sputum purulence could require antibiotics due to concern for possible bacterial infection.

The nurse practitioner is seeing a three day old newborn for her wellness check and her mother is concerned about a new yellow tint to the skin. A bilirubin level is ordered and it results at 13 mg/dl. What is the most appropriate intervention for this newborn?

Correct Answer: Increased feedings and recheck bilirubin only if symptoms worsen or fail to resolve Rationale: When infants present with symptoms of jaundice, fatigue, or poor feeding, a workup for hyperbilirubinemia should be completed. If the result is 12-14 mg/dl, mothers should be encouraged to increase feelings to at least 8-12 times daily. If the bilirubin level is 15 mg/dl or greater, phototherapy will be initiated.

A 47 year old male patient presents to the clinic today with vague complaints of chest pain. Which of the following differential diagnoses would be the most inappropriate for this patient given his chief complaint?

Correct Answer: Inflammatory bowel disease Rationale: Hiatal hernias can sometimes be mistaken for myocardial infarction due to the severity of chest pain they can cause. This is due to the trapping of the stomach above the diaphragm. Pericarditis is known to cause sharp chest pain due to the irritation of the pericardium itself. Peptic ulcer disease is known to cause abdominal pain from the navel to the chest when exacerbations are occurring. Inflammatory bowel disease is the most unlikely diagnosis to cause chest pain, unless a pneumomediastinum was occurring, which is rare.

A 62 year old female patient called the office asking what to do as she just experienced a sudden onset of flashers and floaters in her vision. She states that she is not in any pain, but her vision seems shadowy. Which would be most appropriate for the nurse practitioner to tell the patient to do next?

Correct Answer: Inform her to call 911 as you are worried about retinal detachment Rationale: Retinal detachment often presents as a painless, sudden onset of change in the field of vision such as a curtain or shadow being pulled over it accompanied by flashes and/or floaters in their vision. This is an emergency to prevent permanent vision loss.

An elderly adult male presents with a tremor that occurs during voluntary movement and does not occur at rest. The patient also states that he notices a decrease in the amount of tremors he experiences whenever he drinks alcohol with dinner. Which of the following would not be appropriate to include in his plan of care?

Correct Answer: Initiate a prescription for Carbidopa/Levodopa (Sinemet) Rationale: The cause of essential tremors is oftentimes unknown in most patients. Essential tremors can be distinguished from a Parkinson's disease tremor as they do not occur at rest, but instead occur during voluntary movement. Propranolol is commonly used to treat this condition, and regular physical activity/avoidance of stimulants can help reduce the amount of tremors. Certain medications such as SSRIs (selective serotonin reuptake inhibitors) and antiemetics can make this condition worse. Carbidopa/Levodopa is typically prescribed to those with a true diagnosis of Parkinson's disease.

The nurse practitioner is seeing a 36-weeks-pregnant woman for a routine prenatal visit. It is noted in her chart that she tested positive for Group B Streptococcus at her last visit. The nurse practitioner also notes she has a mild penicillin allergy. She asks what the best plan will be for delivery of her fetus. Which is the most appropriate response?

Correct Answer: Intrapartum IV clindamycin (Cleocin) can be administered and she can deliver safely vaginally Rationale: Group B Streptococcus is tested for between 35 and 37 weeks Gestation. Although Group B Streptococcus is dangerous to a fetus, it is possible to deliver vaginally when IV penicillin is administered intrapartum. If a patient is allergic to penicillin, clindamycin or erythromycin may be used.

A primary care practice is reviewing their policies on how to handle cases of sexual abuse and rape. Which of the following is true regarding mandatory reporting?

Correct Answer: It is mandatory to report both pediatric and elderly cases of sexual assault to law enforcement Rationale: In the United States, the sexual assault of minors or the elderly must be reported to law enforcement. Parental consent is not required for minors who are seeking care after being sexually assaulted.

The nurse practitioner is seeing a 12 year old girl whose mother is concerned about her pierced ear. She reports that she allowed her daughter to get her ear pierced two months ago and it has changed in appearance in the last few weeks. Upon assessment of the area, a raised lesion that extends approximately 0.1 cm beyond the original piercing location is noted. Which of the following is the most likely diagnosis?

Correct Answer: Keloid Rationale: Keloid scars present as raised lesions that extend beyond the area of the original wound. They commonly occur over areas of minor injury, such as piercings. Unfortunately, they continue to enlarge and progress over time and can cause functional impairment depending on the location of the keloid. Hypertrophic scars on the other hand will regress.

A 17 year old male presents to the office after suffering a knee injury at football practice this morning. He claims to have pivoted while trying to catch the ball. He then heard a "popping sound" in his knee and it started to swell immediately. Which diagnostic test can be used to confirm the likely diagnosis?

Correct Answer: Lachman test Rationale: The Lachman test is the most sensitive diagnostic tool to diagnose an anterior crucial ligament (ACL) injury. Once the patient is in the supine position, place one hand behind the tibia and the other hand on the lower thigh. While stabilizing the thigh, bend the knee to a 20 degree angle then pull the leg forward. Excess activity is indicative of an ACL injury. McMurray's test is used to assess an injury of the medial meniscus. Finklestein's test is a diagnostic tool for DeQuervain's tenosynovitis. The valgus stress test of the knee tests for tars in the medial collateral ligament (MCL).

A 56 year old female presents to the clinic today for her annual physical. The nurse practitioner asks if she has any concerns and she reports that she has a "beauty mark" that seems to have grown quite a bit in size over the last several months. Upon examination, it is black with light brown discoloration and ragged borders. Which of the following is the most likely diagnosis?

Correct Answer: Malignant Melanoma Rationale: It is likely that this is malignant melanoma due to the color, increase in size, and the presentation with ragged borders. The ABCDE acronym can be used to evaluate skin lesions for possible malignancy suspicion (A-Asymmetry, B-Border Irregularity, C-Color Variation, D-Diameter >6mm, and E-Evolving/Elevation)

All of the following are possible treatment options for a diagnosis of seasonal affective disorder (SAD) except:

Correct Answer: Melatonin 10mg every night Rationale: For patients with seasonal affective disorder (SAD), first-line treatment includes light therapy, antidepressants such as an SSRI, and incorporation of psychotherapy such as cognitive behavioral therapy (CBT). Experts also suggest supportive interventions along with first-line treatments like having healthy sleep hygiene and for patients to exercise outside, even on cloudy days. However, melatonin is not part of the recommended treatment regiment for SAD.

A primary care provider is reviewing recent guidelines regarding acetaminophen (Tylenol) administration. When reviewing the protocol for acetaminophen (Tylenol) toxicity, the nurse practitioner recalls that the proper antidote is:

Correct Answer: N-acetylcysteine Rationale: N-acetylcysteine is the antidote used for acetaminophen (Tylenol) toxicity. The current recommended maximum daily dose is 3 grams a day. Vitamin K is the antidote for warfarin (Coumadin). Protamine sulfate (Prosulf) is the antidote for Heparin. Flumazenil (Romazicon) is the antidote for benzodiazepines.

A patient's chart notes that they had a thumb spica cast last year. What injury is this cast typically used for?

Correct Answer: Navicular fracture Rationale: A thumb spica cast is used to stabilize the thumb after an acute injury such as a scaphoid fracture, also known as a navicular fracture. The cast is generally worn for up to 12 weeks, with serial 2-week x-rays being done to track the healing of the bone. Home exercise programs may be used in conjunction with the cast.

A new patient presents to the office for an initial visit, and states that they are a member of The Church of Jesus Chris of Latter-Day Saints. Which of the following is in alignment with what the members of this community observe?

Correct Answer: No hot tea or coffee Rationale: The Church of Jesus Chris of Latter-Day Saints is home to members of the Mormon faith. They observe strict purity laws such as no alcoholic beverages or tobacco, no hot tea or coffee, and abstaining fromsexual intercourse until marriage. The Sabbath is a recognized say in the Jewish faith, while Lent is a Catholic holiday season.

A father brings his 11-month-old infant into a flu vaccine clinic. He states they are traveling for a vacation in 2 weeks and is wondering if his son could get his varicella vaccine before they go as they will be around other unvaccinated children. Which of the following is the most appropriate response?

Correct Answer: No, the varicella vaccine can only be given to children 12 months and older Rationale: The varicella vaccine is a primary prevention method to avoid chicken pox and shingles. It is given as a 2-dose series starting at age 12 months old. As it is a live vaccine, it should not be administered before a patient is 12 months old. Pregnant women and immunocompromised people should also not receive live vaccines like varicella. Clarifying note: There are technically certain circumstances that live vaccines can be given before 12 months old, but board exams will not get into the details of this as it is a relatively new guideline and only for extremely specific travel instances.

A patient is complaining of pain and swelling in the elbow joint. When the joint fluid is tested, it is positive for infection. Which type of bursitis is the most likely differential for this patient?

Correct Answer: Olecranon bursitis Rationale: Bursitis is an inflamed and fluid filled sac between the bones and soft tissue. A subacromial bursitis would be bursitis of the shoulder. The olecranon bursitis affects the elbow. Trochanteric bursitis is bursitis of the hip. Calcaneal bursitis affects the heel. Bursitis is typically diagnosed either from a thorough history and physical or from visualization of the fluid from inside the sac. Often bursitis will resolve with rest, ice, and antiinflammatory agents.

A veterinarian presents with a lesion that he states has worsened over the last week. He states that at first it was just a few small "blistery" bumps on his arm that were pruritic like his atopic dermatitis flares. He tells you the lesion did become a little ulcerated, but it didn't hurt and he attributed it to his scratching. He finally decided to be seen as the lesion is starting to get black in the center and it is worrying him. Which of the following treatment plans do you recommend?

Correct Answer: Oral doxycycline (Vibramycin) twice a day for 10 days because you suspect infection from Bacillus anthracis bacteria Rationale: This patient is presenting with signs of cutaneous anthrax. Cutaneous anthrax often starts as small itchy bumps, and progresses to a painless ulcer which is blackened in the center and scabbed over. It is most common to see cutaneous anthrax in patients who work closely with animals such as farmers and veterinarians that work with cattle animals. Our preferred treatment is typically doxycycline or ciprofloxacin for 7-10 days for cutaneous anthrax.

A 42 year old male patient presents to the clinic today with concerns about a "constant shadow" in his groin. Upon exam of the patient's groin, an area of darkened hyperpigmentation is present along with thickening of the skin. The patient's past medical history includes obesity, metabolic syndrome, and hyperlipidemia. Which of the following would be most appropriate for the nurse practitioner to include in this patient's plan of care?

Correct Answer: Order a HgbA1C Rationale: This patient's history puts him at a higher risk for diabetes mellitus. The description is typical of acanthosis nigricans, which typically occurs on the back of the neck, armpit and groin. Acanthosis nigricans is indicative of insulin resistance, and therefore a HgbA1c should be ordered for this patient.

A patient who recently immigrated to the United States is at the clinic to get her purified protein derivative (PPD) test evaluated to gain employment clearance. The induration on her arm measures 12 mm and she is feeling well. What is the best next step by the nurse practitioner?

Correct Answer: Order a chest x-ray Rationale: This patient would be considered an immigrant and therefore is above her cutoff for TB skin testing. Anything ≥10 is considered positive. Since this patient is feeling well and is not complaining of any symptoms, the next best step would be to order a chest x-ray to assess for latent tuberculosis. It would also be important to note if the patient has received the BCG vaccine in the past.

A 71-year-old male presents for follow up after recently being treated for pneumonia. He reports that his shortness of breath has gotten worse and he has noticed speckles of bright red blood when he coughs. He has finished his entire course of antibiotics. The nurse practitioner orders a chest x-ray which notes a small, circumscribed white nodule with volume loss in the left lobe. Which of the following is the next most appropriate action?

Correct Answer: Order a computer tomography (CT) scan of the chest Rationale: The next step in treatment would be getting a low dose CT scan of the chest. It is important to note that even if the x-ray was negative, we would still continue with ordering the CT scan as X-Rays do not always pick up small nodules that can be seen on CT. We want to detect early for a better chance of conservative treatment.

A 65 year old post-menopausal woman presents today with increasing abdominal discomfort and pelvic pain. The nurse practitioner performs a pelvic and vaginal exam, and is able to palpate the patient's right ovary. Which of the following would be the most appropriate plan of care for this patient?

Correct Answer: Order a intravaginal ultrasound as soon as possible Rationale: A palpable ovary in a post-menopausal woman should always be considered an abnormal finding and possibly indicative of ovarian cancer. Therefore, an intravaginal ultrasound should be ordered and the patient should also likely be referred to their gynecologist for further evaluation. A colposcopy is performed for abnormal pap smear results, and Premarin is a drug used to treat hot flashes in menopausal women.

A 26-year-old female patient comes into the clinic wanting the Copper-T intrauterine device (IUD) placed. The patient states that she has not had sex in the last 10 days and does not have any symptoms of an infection. What should the nurse practitioner do next?

Correct Answer: Order a urine pregnancy test Rationale: Although the patient states that she has not had sexual intercourse in the past week, there is still a chance that they could be pregnant, so a urine pregnancy test should be ordered next. There is no need to perform a breast exam during this visit. The intrauterine device (IUD) should not be placed without a negative urine pregnancy test. A vaginal swab should be obtained if the patient presented with symptoms of an infection.

The nurse practitioner is reviewing the urinalysis results of a young adult patient with a suspected urinary tract infection (UTI). She does not have a frequent history of UTI's and is noted to be allergic to penicillins. It is noted in the sample that a multitude of squamous epithelial cells are present. Which of the following is the most appropriate treatment plan for this patient?

Correct Answer: Order an additional urinalysis to review Rationale: Squamous epithelial cells in the patient's urine are oftentimes indicative of the sample being contaminated by cells in the patient's vaginal opening. Due to this possible contamination, a second urinalysis needs to be performed prior to deciding a possible empiric treatment plan.

A 27 year old female patient presents to the clinic requesting a TB skin test for her new employment as substitute teacher. The patient's medical history includes being 28 weeks pregnant as well as generalized anxiety disorder. Which of the following would be the most appropriate plan of care for this patient?

Correct Answer: Order the TB skin test as requested Rationale: According to the CDC, TB skin testing is a safe method of screening for TB during any point of pregnancy. WHile the QuantiFERON-TB Gold blood test can also be utilized, it is not due to any safety concerns associated with the regular TB skin testing.

Which two of the following tests, when positive, are indicative of carpal tunnel syndrome? Select the two answer choices that apply.

Correct Answer: Pain is elicited by tapping the anterior aspect of affected wrist / Pain is elicited when the flexed wrists are pushed together Rationale: Carpal tunnel syndrome is due to median nerve compression, typically caused by repetitive wrist movements like typing. Common risk factors include pregnancy, being female, hypothyroidism, obesity, and rheumatoid arthritis. Tinel's sign is positive if pain is elicited by tapping the anterior aspect of the affected wrist. Phalen's sign, also called the "reverse prayer sign" is positive if pain is elicited when the flexed wrists are pushed together.

A seven year old patient is brought to the clinic for continued right ear pain over the last four days. Which of the following assessment findings would indicate the patient likely has otitis externa?

Correct Answer: Pain upon palpation of the tragus Rationale: Pain upon palpation of the tragus is a classic finding in otitis externa. A bulging and erythematous tympanic membrane would be likely seen with acute otitis media. Otitis media with effusion typically presents with a retracted and full tympanic membrane. A "cauliflower-like" growth is seen more so with a cholesteatoma.

A 72 year old female presents to the clinic after having jaw pain while eating lunch this afternoon. She states she's been especially tired and has had some blurry vision recently and attributes this to a nagging headache near her left eye. After performing a thorough physical assessment, which of the following steps should the nurse practitioner take next?

Correct Answer: Refer to an ophthalmologist for a biopsy Rationale: Jaw claudication, or pain in the jaw with chewing, headache, and blurry vision is suspicious for temporal arteritis. We would want to refer this patient for a temporal artery biopsy as this is the only way to make a definitive diagnosis.

A middle aged patient presents with increasing difficulty breathing and sputum production. The nurse practitioner notes that this patient was recently diagnosed with COPD and he has been getting some relief with the use of his Albuterol inhaler. The nurse practitioner decides to add ipratropium (Atrovent) to the patient's treatment plan. Which of the following conditions in the patient's history would not require that a different medication be used?

Correct Answer: Pancreatitis Rationale: Ipratropium (Atrovent) belongs to a class of drugs called short acting muscarinic antagonists. This drug class is notorious for having anticholinergic side effects, which exacerbate many known conditions. Anticholinergics have a wide array of side effects such as dry mouth, blurry vision, urinary hesitancy and constipation. In BPH, anticholinergics would worsen the symptoms of urinary hesitancy. In acute-angle closure glaucoma, anticholinergics can induce this condition to occur again and it is considered to be a medical emergency. In peptic ulcer disease, anticholinergics slow down the process of gastric emptying, therefore exacerbating any current ulcers. Anticholinergics are often used in cases of pancreatitis due to their ability to slow down the secretion of pancreatic enzymes as well as gastric acid.

All of the following are potential secondary causes of hyperparathyroidism except:

Correct Answer: Parathyroid adenoma Rationale: Secondary hyperparathyroidism occurs when an issue external to the parathyroid is lowering calcium levels. This can be due to chronic kidney disease (especially end stage renal disease), a vitamin D deficiency, or neck radiation. Primary causes of hyperparathyroidism are due to an issue with the parathyroid itself, such as a parathyroid adenoma.

A middle aged adult patient presents for a follow-up as he was recently started on an antidepressant. The patient notes a reduction in symptoms but reports he is having issues with erectile dysfunction. The nurse practitioner knows which of the following medications from the selective serotonin reuptake inhibitor (SSRI) class most commonly causes this issue?

Correct Answer: Paroxetine (Paxil) Rationale: Paroxetine (Paxil) is notorious for issues with both sedation and erectile dysfunction. Therefore, Wellbutrin is commonly prescribed alongside this medication to combat that issue. In addition, Venlafaxine is not an SSRI, and is instead a SNRI.

When a patient presents to your office with all of the signs and symptoms of Meniere's disease, which of the following is the biggest concern for their long term health?

Correct Answer: Permanent loss of hearing for high pitched sounds Rationale: Meniere's disease is a not well understood disease of the inner ear that results in changes in hearing, dizziness, vertigo, and nausea for patients. It can be extremely unpleasant and interfere with quality of life. If it progresses far enough, it can lead to total loss of hearing, which is an irreversible consequence of Meniere's.

The nurse practitioner suspects a patient might have carpal tunnel syndrome as the patient notes pain when he pushes the backs of his flexed wrists together. Which clinical sign is this?

Correct Answer: Phalen's sign Rationale: Phalen's sign, also called the "reverse prayer sign" is positive if pain is elicited when the flexed wrists are pushed together. The proper way to perform this is to have the patient press the backs of their hands together, with flexed wrists, for 60 seconds. If pain or a tingling sensation is elicited, this is a sign of carpal tunnel syndrome.

A patient is very concerned about their recent diagnosis of polycythemia vera. The nurse practitioner decides to provide some further anticipatory guidance for this patient. Which of the following would be inappropriate to educate this patient about?

Correct Answer: Phlebotomy is often initiated until the hematocrit is less than 35%. Rationale: Phlebotomy is actually often initiated until the hematocrit is less than 45%, not 35%. The patient will require lifelong treatment as there is no cure present. Hydroxyurea can be utilized for these patients in order to slow down the production of more red blood cells. Aspirin can also be used to further help thin out the blood.

A middle aged factory worker comes into the clinic with complaints of a dull right heel pain and stiffness. He reports the pain is mostly when he is finished walking at work as he walks up to ten miles per day. He is also having increasing difficulty climbing stairs due to the pain. Which of the following is the most likely differential for this patient?

Correct Answer: Plantar fasciitis Rationale: Plantar fasciitis typically presents as heel pain at the bottom of the heel. The pain commonly appears when the patient stops activity or first thing in the morning. This condition is usually unilateral in nature and is often treated with RICE (Rest, Ice, Compress, Elevate) initially.

The nurse practitioner is assessing an 80-year-old patient brought in by his daughter today for "memory trouble." The patient is administered the Mini Cognitive Assessment Quiz (Mini Cog) and scores a 2. Which of the following is the correct interpretation of this score?

Correct Answer: Positive screen for dementia Rationale: The Mini Cognitive Assessment Quiz (Mini Cog) is a tool which can be used to assess patients for dementia. A negative screen (no dementia) would be 3-5, while a positive screen (positive for dementia) would be 0-2. A score of 0-3 is given for the recall portion of the assessment, while 0 or 2 are the possible scores for the clock drawing test.

Which of the following is the least likely sign or symptom of epiglottitis in children?

Correct Answer: Preauricular adenopathy Rationale: Epiglottitis is a serious medical condition in which the upper airways begin to swell. Some typical exam findings include drooling, strider, and odynopagia. It is more typical to see cervical chain adenopathy develop with this condition.

After suffering injuries from a severe motor vehicle accident, a patient became dependent on narcotics. The patient is wanting to quit and is now researching narcotic anonymous meetings in her area. This is an example of which of the following phases from the "Stages of Change" model?

Correct Answer: Preparation Rationale: When a patient is taking small steps to prepare for the change, it is considered the preparation stage. Precontemplation is when the patient does not want to change or does not see a need to change a behavior. Contemplation is when they are starting to think about the change, but have not taken any concrete steps towards planning for the change. Action is when the patient's plans are taking effect and the patient has taken actual action into the change itself. Maintenance is when this change is sustained overtime, typically at least 6 months.

A 62 year old patient with a history of osteoporosis and diabetes presents for a follow up appointment regarding her heartburn. She has been trying lifestyle modifications as well as calcium carbonate (Tums) for the past few months which has helped a little, but she would like something more effective today. Which of the following would be most appropriate for this patient?

Correct Answer: Prescription for famotidine (Pepcid) Rationale: An H2 blocker like famotidine would be an appropriate medication for this patient. Her GERD symptoms are not being controlled with Tums or lifestyle modifications. Both H2 blockers and proton pump inhibitors (PPI's) can be prescribed for GERD. However, with the patient's history of osteoporosis, an H2 blocker would be a safer, long-term option for this patient.

A patient comes into the clinic thinking that she is pregnant since her period is one week late. The nurse practitioner understands that amenorrhea is an example of what sign of pregnancy?

Correct Answer: Presumptive Rationale: Presumptive signs of pregnancy are subjective things reported by the mother. The patient often reports breast tenderness, nausea and vomiting, kicking, amenorrhea and quickening. Probable signs include a positive urine pregnancy test along with Chadwick and Hegar's sign. Positive signs of pregnancy are clinical signs that objectively prove the patient is pregnant to the provider. Early objective findings are cardiac activity and use of doppler to detect fetal heart rate. Later, the provider palpating the fetus is also a positive sign.

The nurse practitioner recommends her patient with an increased ASCVD risk score take a low-dose aspirin daily. Which level of prevention is this?

Correct Answer: Primary prevention Rationale: Taking a low dose aspirin to help prevent cardiac events in the future in high risk patients is an example of primary prevention. Primary prevention is about preventing disease. Secondary prevention is related to screening/early detection of a disease. Tertiary prevention is about treating and reducing complications of a disease.

A 23-year-old female comes in complaining of ongoing migraines. She states she has had 5 migraines in the last month and attempts at identifying and decreasing triggers have not been successful. She has a past medical history of polycystic ovary syndrome, mild asthma, and rheumatoid arthritis. Which of the following would be an inappropriate choice of prophylactic medication for this patient?

Correct Answer: Propranolol (Inderal) Rationale: Putting this patient on a prophylactic migraine medication would be appropriate due to her ongoing migraines. While all of these are viable prophylactic medication options, the patient's history of mild asthma would not make her an ideal candidate for a beta blocker like propranolol. Amitriptyline, a tricyclic antidepressant, can be taken at bedtime to prevent migraines. Anticonvulsants like topiramate may also be used, as well as Calcitonin gene-related peptide (CGRP) medications such as Erenumab.

Which of the following drugs are not ideal to prescribe to the elderly specifically due to their long half life? Select all that apply by selecting four of the following answer choices.

Correct Answer: Prozac / Digoxin / Tegretol / Amiodarone Rationale: The half-life of fluoxetine (Prozac) is approximately 48-96 hours. The half-life of digoxin (Lanoxin) is approximately 36-48 hours. The half life of carbamazepine (Tegretol) is roughly 24-72 hours. Amiodarone (Cordarone) has an extended half life of anywhere between 1-6 months. Diphenhydramine (Benadryl) and sumatriptan (Imitrex) are generally avoided in the elderly. However, they both have relatively short half lives.

An 11-year-old patient comes into the office complaining of a severe sore throat, malaise, and a headache. Upon exam, you see a dark pink pharynx with some petechiae on the hard palate. The remainder of the exam is benign. What would be the most appropriate test to initially order for this patient?

Correct Answer: Rapid antigen detection test Rationale: This patient is most likely exhibiting signs of an acute streptococcal pharyngitis infection. The key findings here are the dark pink pharynx with petechiae on the hard palate (also known as palatine petechiae) as well as his benign exam otherwise. A rapid antigen detection test, or "rapid strep" test is the best option for this patient.

A 70 year old male patient is at the office for an annual physical exam. He tells the nurse practitioner that his wife had noticed brown lesions on his back that resembled warts. After diagnosing the patient with seborrheic keratosis, which of the following would be most appropriate to tell the patient?

Correct Answer: Reassure the patient that these are benign growths and they are a common finding in the elderly Rationale: Seborrheic keratosis is benign, common skin finding in the elderly. This presents as wart-like growths or "pasted on" lesions that range in color from tan, brown, to black and most commonly found on the back. For this diagnosis, the nurse practitioner can reassure the patient that the condition is benign and will cause no harm. If bothersome, they can be removed for the patient.

The nurse practitioner is seeing a 22 year old male who is a competitive baseball player. After discussing magnetic resonance imaging (MRI) results indicative of a rotator cuff tear, the patient asks how this can happen. All of the following are correct statements except:

Correct Answer: Recurrent dislocation of the shoulder joint Rationale: In this patient, a rotator cuff injury is caused by a repetitive throwing motion which causes stress on the rotator cuff muscles and tendons overtime. Athletes most commonly affected by this are tennis and baseball players. Other causes include falling with hands extended forward, and sudden pulling motion of the shoulder.

The nurse practitioner is reviewing the pap-smear results for a 21-year old patient. The report notes that there are atypical squamous cells and can not exclude high-grade squamous intraepithelial cells (ASC-H). Which of the following is the best plan of action for this patient?

Correct Answer: Refer for colposcopy Rationale: When cytology results come back as atypical squamous cells that can not exclude high-grade intraepithelial changes, regardless of age, we refer for a colposcopy. This is because high-grade changes in the intraepithelial cells can indicate pre-cancerous cells. A colposcopy can visualize the cervix and collect biopsies if needed, which can help diagnose early cervical cancer, which is key to treatment and survival.

A 59 year old female patient presents to the clinic with continued throat pain and hoarseness over the last five weeks. Her past medical history includes smoking, COPD, hypertension and osteoarthritis. What should the nurse practitioner anticipate ordering next?

Correct Answer: Refer the patient to an ENT for further evaluation Rationale: Due to the patient's history of smoking and complaints of unresolving throat pain along with hoarseness, there is concern for the development of laryngeal cancer. Hoarseness lasting longer than three weeks without an obvious cause is a red flag. She should schedule a visit with ENT for further evaluation of her symptoms.

A 66-year-old female comes with her husband with complaints of a headache. She reports that the headache started last night and it is unlike any headache that she has ever had. Her husband tells you that he is worried because she has also been having periods of irritability and aggression which is unusual for her. What should the nurse practitioner do next?

Correct Answer: Refer to the emergency department for a computed tomography (CT) scan of the head Rationale: When a patient presents with "the worst headache of their entire life," it is important that the patient is referred emergently for further imaging. The provider should be concerned for stroke or in this instance, a brain tumor due to her associated personality changes.

A woman brings her 92 year old father into the office today with concerns for Parkinson's disease. She notes he recently started developing a right hand tremor at rest and his right arm "just doesn't move like it used to". The patient states that his arm feels very stiff, but since he has little to no pain it is not overly bothersome to him. He denies any difficulty performing his day to day activities. What is the best treatment plan to initiate at today's visit?

Correct Answer: Referral to palliative care Rationale: The most appropriate plan for this patient at this time would be the initiation of palliative care. Although he is exhibiting very mild symptoms at this time, palliative care should always be initiated at the time of diagnosis when possible. Not only does palliative care help with physical needs and symptom management, they provide a lot of psychosocial care as well which is especially important upon disease diagnosis. Carbidopa/Levodopa (Cinemet) may be imitated as the disease progresses, however is not appropriate at this time for a few reasons; this medication works best at treating bradykinesia which this patient is not yet experiencing and approximately five years after drug initiation, it will likely no longer be effective for the patient. For these reasons, it is important to only begin this drug when absolutely necessary.

A sexually active 20-year-old patient comes into your office for evaluation. She states she would like to be tested for all possible sexually transmitted diseases as she is unsure of her last partner's status. Which of the following would be an inappropriate test to order?

Correct Answer: Reflex human papilloma virus (HPV) DNA testing Rationale: All of the above choices are reasonable tests to run when a patient, regardless of age, comes in for sexually transmitted infection (STI) testing. Rapid plasma reagin tests can test for syphilis, while the nucleic acid amplification test can be used to detect gonorrhea and chlamydia. The HIV-1/HIV-2 antibody antigen assay test is an extremely specific test to determine HIV infection status. However, due to her age, this patient does not need HPV testing. This is because younger females typically clear any infection so quickly, it is not of a concern. Routine HPv testing begins at age 21 in the presence of an abnormal pap-smear.

A 17 year old athlete presents to the clinic today for her yearly sports physical. She notes that she has recently been wearing a fitness tracker and has noted some fluctuations in her heart rate. Due to significant cardiac history in her family history, you decide to order an electrocardiogram (ECG) for this patient. While connected to the electrocardiogram (ECG), it is noted that the R-R interval is shortened when the patient breathes in, and prolonged when the patient breathes out. What is the most likely diagnosis for this patient?

Correct Answer: Respiratory sinus arrhythmia Rationale: If a patient experiences an increased heart rate with inspiration and decreased heart rate with expiration, this is known as respiratory sinus arrhythmia. Patients with this are typically asymptomatic and the diagnosis can be confirmed with an electrocardiogram (ECG). Education should be provided to the patient that this is a benign condition and requires no treatment or further evaluation.

A 40-year-old female patient comes into the office today complaining of slow onset fatigue, fever, and generalized body aches. Upon exam, she has large sausage-shaped joints and her distal interphalangeal joints are hyperextended, with flexion in the proximal interphalangeal joints. Which of the following would be the most likely diagnosis for this patient?

Correct Answer: Rheumatoid arthritis Rationale: Rheumatoid arthritis is a systemic disease that is more common in women. It is identified by inflammation in multiple joints, generalized fatigue, and early morning stiffness with joint inflammation. It is typically treated by the Disease-modifying antirheumatic drugs (DMARDs) as it is an autoimmune disorder. It is also imperative to refer these patients to rheumatology for management.

A 27 year old patient presents to the clinic complaining of worsening nasal congestion after being diagnosed with acute bacterial sinusitis 3 weeks ago. Since finishing his antibiotic he feels much better overall, but everytime he stops using his oxymetazoline (Afrin) nasal decongestant spray, his sinus congestion gets worse. He is requesting another course of antibiotics to help get rid of this sinus infection. What should the provider diagnose this patient with?

Correct Answer: Rhinitis medicamentosa Rationale: Rhinitis meidcamentosa, or drug-induced rebound congestion, can happen after use of nasal decongestants like Afrin. These nasal decongestants should ideally not be used for greater than 3-5 days because of this risk.

A 27 year old patient presents to the clinic complaining of worsening nasal congestion after being diagnosed with sinusitis 3 weeks ago. Since finishing his antibiotic he feels much better overall, but everytime he stops using his oxymetazoline (Afrin) nasal decongestant spray, his sinus congestion gets worse. He is requesting another course of antibiotics to help get rid of this sinus infection. What should the provider educate this patient about?

Correct Answer: Rhinitis medicamentosa Rationale: This patient likely has rhinitis medicamentosa, or rebound congestion, due to his prolonged use of his Afrin nasal decongestant. The patient needs to be instructed that nasal decongestants should not be used for more than 3-5 days and that he needs to stop the medication. He can slowly wean off of it by only using it once a day and then completely to help the withdrawal congestion. Additionally, systemic decongestants, nasal corticosteroids, nasal irrigation, and possibly a small course of oral corticosteroids can help the withdrawal symptoms.

A middle aged woman presents to the office today with a new facial rash. Upon examination, the nurse practitioner notes erythema and telangiectasias on both of her cheeks and the nose. Overall, the facial skin appears dry and there is mild edema of the cheeks. She denies difficulty breathing, swallowing, and any throat or tongue swelling but endorses a burning sensation. Which of the following diagnoses should be considered based on this assessment? Select all that apply by choosing two of the following answer choices.

Correct Answer: Rosacea / Basal cell carcinoma Rationale: Both rosacea and basal cell carcinoma should be considered for this patient. Rosacea presents as an erythematous facial rash that does not spare the nose. Dry skin, facial edema, burning/stinging, and telangiectasia are also seen with rosacea. Treatment for this would include topical flagyl gel. Basal cell carcinoma should be considered as the face is frequently exposed to the sun, making it a common area for basal cell carcinoma to occur. In addition, telangiectasia is a classic sign of basal cell carcinoma. Although a burning sensation is present, shingles is unlikely as it occurs along a dermatome and presents as vesicles.

A 77 year old female presents to the office with diminished central vision and line distortion. She notes this came on very gradually and denies eye pain. She has a history of gout, osteoporosis, and hypertension. What would be the most appropriate intervention to discuss with this patient?

Correct Answer: Selecting large print books Rationale: This patient is presenting with signs and symptoms of macular degeneration. Since macular degeneration is caused by damage to the retina, corrective lenses are not going to provide benefit for this patient. However, larger print may make it easier for the patient to read. As well, studies have shown that uncontrolled hypertension is directly related to macular degeneration, so strict blood pressure control will be important for this patient, however there is not sufficient evidence to support the use of statin therapy in those with this diagnosis. Finally, lens replacement surgery is used to treat cataracts, not macular degeneration.

A woman at 18 weeks gestation obtains a urine sample as part of a routine prenatal visit. She denies any pain or burning with urination, but there are positive leukocytes and nitrites present on the urinalysis. Which of the following is true regarding treatment?

Correct Answer: She can be treated with Amoxicillin (Amoxil) for her urinary tract infection Rationale: Pregnant patients with urinary tract infections are more likely to develop complications such as pyelonephritis which can lead to preterm delivery and low birth weight infants. Because of this, it is recommended to treat asymptomatic bacteriuria in pregnant women with antibiotics. Amoxicillin is safe throughout all trimesters of pregnancy and can be used for urinary tract infections as long as the bacteria is susceptible. Antibiotics such as Keflex (Cephalexin) and Macrobid (Nitrofurantoin) can also be used during particular trimesters, but fluoroquinolones like Ciprofloxacin should be avoided in pregnant women.

A 46-year-old woman comes in for her annual well-woman exam and a pap smear. She states one of her friends recently was diagnosed with breast cancer and is wondering what she can do for screening tests. She states no one in her family has had breast cancer to her knowledge, but her father had skin and colon cancer. What is the best recommendation for breast cancer screening with this patient?

Correct Answer: She should begin getting mammograms at least every year Rationale: Mammograms for patients who are at a regular risk for breast cancer should begin at age 45 and be repeated at least every year, according to the American Cancer Society.After age 54, it is safe to move to every other year, although every year should still be encouraged. This does not apply to patients with a family history of breast cancer as they may need screening earlier. This patient should also have a colonoscopy done, but she additionally should have a mammogram. BRCA 1 and 2 testing is recommended by The American Society of Clinical Oncology for those with familial history of breast cancer and/or a personal history of epithelial ovarian cancer.

You see a patient in the clinic today who complains of pain and swelling around their mouth that occasionally radiates down into the neck. The pain worsens just before meals and with eating. She notes that recently, she has experienced an unusually foul taste in her mouth that does not seem to improve despite the use of mouth rinses. What is the most likely diagnosis?

Correct Answer: Sialolithiasis Rationale: Sialolithiasis, or a salivary gland stone, can cause symptoms including pain and swelling of the face, jaw, and neck that worsens before or with eating when saliva production increases. It can also cause a foul taste or unusually dry mouth. These can be treated with hydration, moist heat applications, and massaging the area. If symptoms fail to improve, occur frequently, or there is suspicion for a salivary gland tumor, the patient should be referred out.

A 9 year old male comes to the office accompanied by his mother to follow up on a recent visit to the emergency room (ER). The patient's mother had brought him to the ER because he was struggling to breathe after playing soccer on a hot day. She tells you that she is confused because he was diagnosed with an asthma exacerbation in the ER, but he has never been diagnosed with this before. Which diagnostic tool is used to confirm the diagnosis of asthma?

Correct Answer: Spirometry Rationale: Spirometry is the primary diagnostic tool for both asthma and chronic obstructive pulmonary disease (COPD) because it is a true measure of lung function. Asthma staging is determined by forced expiratory volume in 1 second (FEV1) which measures the air forced in and out of the patient's lungs in 1 second. Intermittent and mild persistent asthma have an FEV1 >80%. Moderate persistent asthma has an FEV1 60-80%. Severe persistent asthma has an FEV1 >60%.

Which of the following wet mount descriptions would make the nurse practitioner suspect bacterial vaginosis?

Correct Answer: Squamous epithelial cells with blurred margins that are coated with bacteria Rationale: Bacterial vaginosis would reveal a wet mount with clue cells, or squamous epithelial cells with blurred margins coated with bacteria, that may resemble peppered eggs. A wet mount that reveals too many white blood cells to count is indicative of an infection, such as chlamydia or gonorrhea. The presence of flagella may indicate trichomoniasis.

The nurse practitioner is starting a 38-year-old male patient on citalopram (Celexa) for his depression. Which of the following alternative treatments can not be used while taking this drug?

Correct Answer: St. John's Wort Rationale: St. John's Wort is an SSRI (selective serotonin reuptake inhibitor) alternative that can cause serotonin syndrome when given in conjunction with other SSRIs, like citalopram. It is important to always check current medications and alternative therapies patients might be on to be sure that they will not interfere with new drugs.

A patient with hypertension has been inconsistent with taking his anti-hypertensive medication. The nurse practitioner should educate the patient that this tertiary prevention method is important to prevent which of the following complications? Select all that apply by choosing three of the following answer choices.

Correct Answer: Stroke / Memory loss / Kidney disease Rationale: Diabetes and hypertension are the two leading causes of chronic kidney disease. The increased pressure against the vessels in patients with hypertension also puts them at an increased risk for hemorrhagic strokes. Hypertension can lead to changes within the brain causing memory loss and dementia. Hypertension can also cause coronary artery disease and lead to changes in the eyes like AV nicking. Hypertension is not correlated with causing diabetes, hepatitis, or neuropathies.

All of the following are true regarding quasi-experimental design except:

Correct Answer: Subjects are randomly assigned to treatment groups and control groups within the study. Rationale: The difference between a true experiment and a quasi-experimental design is that randomization of the subjects amongst various treatment groups is not a requirement of the study.

After reviewing magnetic resonance imaging (MRI) results revealing a partial rotator cuff tear, the nurse practitioner discusses all of the following treatment options with the patient except:

Correct Answer: Surgical repair Rationale: Majority of partial rotator cuff tears can be treated with physical therapy to strengthen the scapular and rotator cuff muscles. Oral nonsteroidal anti-inflammatory drugs (NSAIDs) and corticosteroid injections performed by an orthopedist. Surgical intervention is reserved for full-thickness rotator cuff injuries that tend to not heal well. Overtime, a degenerative process occurs where muscle is replaced by fat causing fatty infiltration of the muscle.

A male patient presents with bilateral pain and swelling of his scrotum. Which of the following findings would make the nurse practitioner suspect a potential diagnosis of orchitis?

Correct Answer: Swelling near his left cheek and neck Rationale: Orchitis is often preceded by a mumps infection and the development of parotitis. With administration of the MMR vaccine, cases of orchitis are much more rare. An absent cremasteric reflex could indicate testicular torsion in which the nurse practitioner would need to emergently refer the patient. RBC casts seen on a urinalysis would indicate possible glomerulonephritis. A asymmetrical, firm, fixed nodule (typically painless) is suspicious for testicular cancer.

Which of the following tests would be the most sensitive for confirming a possible diagnosis of gout?

Correct Answer: Synovial fluid aspiration Rationale: The most sensitive test to confirm a diagnosis of gout is a synovial fluid aspiration from the affected joint and visualization of the crystals in the fluid. A serum uric acid is a more cost effective and less invasive test that can be performed for initial suspicions of gout. However, it is possible for the patient to be experiencing an exacerbation and have a normal uric acid level. X-rays can help rule out other conditions such as osteoarthritis or rheumatoid arthritis. An ultrasound could be used to identify gout, but is not as specific as the synovial fluid aspiration.

Which of the following headaches typically presents as bilateral in nature?

Correct Answer: Tension headache Rationale: Migraines, cluster headaches and temporal arteritis most commonly present as unilateral headaches. Tension headaches are typically described as a diffuse, bilateral pain across the entire head.

A provider has diagnosed a patient with a history significant for falls with benign prostatic hyperplasia. Which medication should the provider prescribe?

Correct Answer: Terazosin (Hyrtin) Rationale: First line treatment for benign prostatic hypertrophy is generally the alpha blocker class which includes terazosin (Hytrin) and tamsulosin (Flomax). These have a first time dose effect of orthostatic hypotension and this patient should be educated to take the first dose at night when he goes to bed to lower the risk of falls. A 5-alpha reductase inhibitor like finasteride (Proscar) could be a good second line option. These work by reducing the size of the prostate over time, but may take up to 6 months to see full effect.Tadalafil (Cialis) is used for erectile dysfunction. Alendronate (Fosamax) is used for osteoporosis.

Which of the following statements is incorrect in regards to evaluating an anterior cruciate ligament tear?

Correct Answer: The anterior drawer test is more sensitive than the lachman test. Rationale: The lachman test is more sensitive than the anterior drawer test for an ACL tear. The lachman test is performed by moving the hip to 45 degrees and the knee to 90 degrees. Then, the examiner jerks the knee forward. If there is too much range of motion, the knee is no longer being stabilized appropriately by the ACL.

A patient presents with intense pruritus. In which of the following scenarios would the nurse practitioner treat the entire household?

Correct Answer: The patient has numerous lesions around the waistline that are itchiest at night Rationale: Pruritic lesions around the waistline, in folds of skin, or webs of fingers/toes that are itchy at night are suspicious of scabies. Scabies is an infestation of the Sarcoptes scabiei mite. They spread readily from person to person, especially through soft materials like bedding and clothing. Entire households should be treated when one member has a confirmed case, with a repeat treatment in 7 days to make sure all mites have been killed. Additionally, all bedding and clothing should be washed in hot water.Varicella would present as pink itchy blisters and there is no treatment outside of symptomatic care for these patients. Honey-colored crusts are seen with impetigo. While impetigo can be extremely contagious, only those with symptoms need treatment. Ideally they will keep the lesions covered and practice good hand hygiene to prevent spreading the lesions. Erythema migrans can happen after a tick bite and only the infected individual needs treatment.

A 70-year-old patient presents to the clinic complaining of feeling like when people talk to him, their voices are muffled in his right ear. The nurse practitioner conducts a Rinne test, which is normal. A weber test is then performed, which lateralizes to his left ear. Which of the following is the correct interpretation of these results?

Correct Answer: The patient has sensorineural hearing loss in their right ear Rationale: The Weber test is performed by placing the tuning fork midline on the patient's forehead. In a normal result, the sound does not lateralize (sound louder) in either ear. When a patient has conductive hearing loss, the sound lateralizes to the affected ear. When a patient has sensorineural hearing loss, the sound lateralizes to the unaffected ear. In the case of this patient, the sound lateralized to the unaffected ear as he had a complaint about his right ear and the sound lateralized to the left ear. This, combined with the normal Rinne test, indicates that the patient is experiencing sensorineural hearing loss.

A female patient comes in complaining of vaginal discharge that has a milky-white appearance for the last few days. She endorses she is in a heterosexual monogamous relationship and rarely uses condoms. She takes oral contraceptives and states she never misses active pills. The nurse practitioner performs a wet-prep/smear, and it comes back negative for WBC (white blood cells), and positive for clue cells. Which of the following is the most appropriate plan of care for this patient?

Correct Answer: The positive clue cells and negative WBCs are indicative of bacterial vaginosis and only she will need to be treated as it is not contagious Rationale: Clue cells are a hallmark sign of BV, or bacterial vaginosis. Along with the milky white discharge, this is a classic presentation. BV is not contagious, and is treated with oral Flagyl twice a day x 7 days. Only the patient will need to be treated, not her partner. She should be educated to not douche as this is a high risk factor for BV.

Which of the following statements is incorrect regarding "incident to billing"?

Correct Answer: This allows for the nurse practitioner to be reimbursed at the physician rate of 100% instead of the nurse practitioner rate of 75% Rationale: Incident to billing allows the nurse practitioner to be reimbursed at the physician rate of 100%. The usual reimbursement by Medicare for nurse practitioners is 85%, not 75%. There are specific instances when incident to billing can be utilized which include: the physician must be in the office at the time, the physician has to have seen the patient before/initiated a treatment plan, and the nurse practitioner can only see the patient for what they were previously seen for by the physician.

When assessing a potential knee injury, a patient is instructed to lay face down on the exam table. The nurse practitioner flexes the patient's right knee to ninety degrees and then places their knee on the back of the patient's thigh. Once stable, the provider compresses down on the patient's foot, and rotates the tibia medially and laterally. The patient reports pain with this maneuver. What does this tell you?

Correct Answer: This is a positive Apley's Grind test and indicates a meniscus injury Rationale: The test being described is the Apley Grind test, which is indicative of a meniscus injury. The Posterior Drawer test, used to assess for posterior cruciate ligament tears, is done by having the patient flex the knee to 90 degrees while lying supine and attempting to move the lower leg posteriorly. The Valgus Stress Test, done to assess for medial collateral ligament injuries, is done by placing one hand on the ankle and one over the femur while attempting to externally rotate the tibia. Finally, the Lachman test is performed by placing one hand over the femur and the other over the tibia. If the tibia is able to be moved anteriorly, this indicates damage to the anterior cruciate ligament.

An 84 year old male patient presents to the clinic with his adult daughter as she reports that he has been having more intermittent confusion recently. The nurse practitioner suspects a possible differential of dementia, and performs the MMSE (Mini-Mental State Exam). The patient scores 19 on the exam. Which of the following is the correct interpretation of these results?

Correct Answer: This score is indicative of a possible moderate dementia Rationale: The maximum possible score on the MMSE is 30 points. A score of greater than 25 indicates dementia is not a likely differential. 20-24 is indicative of mild dementia, 13-20 indicates moderate dementia, and scores of less than 12 indicate severe dementia.

A 17-week-pregnant woman comes into the clinic for evaluation due to new onset vaginal bleeding. The nurse practitioner performs a cervical exam and notes that the cervical os has remained closed. What would be the appropriate diagnosis for this patient?

Correct Answer: Threatened miscarriage Rationale: A threatened miscarriage or abortion occurs before 20 weeks gestation and is characterized by vaginal bleeding, but a cervical os remaining closed. In many cases it is possible to stop the miscarriage and continue the pregnancy. It is not inevitable, where the cervical os opens and the process is unable to be stopped. A complete miscarriage or abortion happens when the placenta and fetus are completely expelled. Placenta previa is generally present >20 weeks and presents as painless vaginal bleeding.

A 19 year old female patient presents with worsening migraines over the last two months. She has tried preventative Metoprolol (Lopressor) in the past without success and is now looking for an alternative treatment. Her past medical history is significant for bulimia recovery as well as major depressive disorder. Which of the following preventative migraine treatments would be the least appropriate to prescribe to this patient?

Correct Answer: Topiramate (Topamax) Rationale: Topiramate would be the least desirable preventative option for this patient due to her history of having an eating disorder. Topiramate has the potential for weight loss to occur, which could have negative impacts upon her eating disorder recovery process. In addition, the question asked about preventative treatments, and the drugs listed from the triptan drug class are considered to be abortive treatments instead.

A 78 year old patient presents with new complaints of an aggravating sensation in his legs that worsens at rest. After investigating further, you begin to suspect a possible diagnosis of restless leg syndrome. Which of the following medications would be inappropriate to initially prescribe for symptom relief in this patient?

Correct Answer: Ultram (Tramadol) Rationale: Pain medication should only be initiated in those with severe symptoms who have failed previous treatment. The basis of care is to prescribe medications that increase dopamine such as Pramipexole, Ropinirole, or Rotigotine. These medications are dopamine agonists.

Your patient presents today with complaints of visual loss and eye pain. Upon examination, you note inflammation, erythema between the cornea and the sclera, and constricted pupils. What would be the most appropriate plan for this patient?

Correct Answer: Urgent ophthalmology referral as they have iritis Rationale: This patient is presenting with classic signs and symptoms of iritis. Iritis requires an urgent referral to ophthalmology as this patient will need to be started on steroid eye drops to treat and prevent long term complications such as permanent blindness.

A 6 year old patient presents with urinary complaints today to the clinic. His mother reports that they recently moved out of their house due to a divorce and the child has been alternating between staying one week with her and one week with his father. He has been having urinary accidents throughout the day, which is unlike him. She notes he will walk around the house holding his stomach as if he is in pain and is always asking to use the bathroom, but seems scared to start urinating once she takes him. What is the most likely diagnosis?

Correct Answer: Urinary tract infection Rationale: Although this patient may show signs of regression with the recent divorce and changes in his day to day routine, he is showing additional symptoms such as abdominal pain, urinary frequency, and likely painful urination which are more indicative of a urinary tract infection. A urinalysis should be performed in this patient and antibiotics prescribed if indicated.

An elderly male patient presents to the clinic for a follow-up after recently having an ischemic stroke. His daughter brings him to the clinic as he has a residual deficit of Wernicke's aphasia. Which of the following statements is incorrect in regards to this condition?

Correct Answer: Wernicke's aphasia presents as an expressive aphasia, which means that the patient has difficulty producing speech. Rationale: Wernicke's aphasia is common after ischemic strokes. When this area of the brain is damaged, a receptive aphasia develops in which the patient can no longer fully comprehend language. Speech and language therapy is the mainstay of treatment for aphasia. When there is damage to Broca's area instead, an expressive aphasia is developed, where the patient has trouble expressing language.

A patient with a history of recurrent otitis media presents with a complaint of hearing loss in their right ear. On the otoscopic exam there is a white, cauliflower looking growth present. Which of the following assessment findings is consistent with this diagnosis?

Correct Answer: When performing the Rinne test on the affected side, BC>AC Rationale: This patient is presenting with a cholesteatoma in their right ear which causes conductive hearing loss. A normal Weber test would have no lateralization. In a conductive hearing loss, the sound would lateralize to the affected ear, in this case the patient's right ear. In a sensorineural hearing loss, the Weber test would result in sound lateralized to the unaffected ear. A normal Rinne test is AC>BC (air conduction > bone conduction). In a conductive hearing loss, BC>AC (bone conduction > air conduction).

How would the nurse practitioner assess cranial nerve IX?

Correct Answer: While assessing the throat, ask the patient to say "ah" Rationale: The glossopharyngeal nerve (CN IX) has both sensory and motor effects and innervates the tongue and the throat. Cranial nerves IX and X are often tested together and can be observed by watching the uvula rise and fall with phonation. The weber test would be assessing the acoustic (or vestibulocochlear, CN VIII) nerve. Asking a patient to raise their eyebrows or puff their cheek tests the facial nerve (CN VII). Asking a patient to stick out their tongue and move it side to side assess the hypoglossal nerve (CN XII) which is responsible for the movement of the tongue itself.

A pregnant patient is interested in getting the Copper-T intrauterine device (IUD) for contraception after delivery and would like to learn more information about this method. Which of the following statements is appropriate to tell the patient?

Correct Answer: You can have this placed within the first 48 hours after birth Rationale: The Copper-T IUD is a hormone free intrauterine device that is effective for up to 10 years. After it is placed, the patient should be instructed to check for the strings every month to verify placement. Since it is hormone free, this is safe for breastfeeding mothers. The Levonorgestrel IUD lasts for 3-6 years and is slightly more effective than the Copper-T IUD. The Levonorgestrel IUD has a typical use failure rate of up to 0.4%, while the Copper IUD's typical use failure rate is 0.8%. After delivery, an IUD can be placed within the first 48 hours after birth. After that window, they have to wait until 4-6 weeks postpartum.


Set pelajaran terkait

College General Psychology: Module 3 Exam

View Set

The Story Of A Vision Study Sync

View Set

Computer Science IB HL: Topic 1 - System Fundamentals

View Set

US History: Chapter 34 America in World War II

View Set

Lippincott Chapter 13: Fluid & Electrolytes

View Set